Retired NPTE Form 3.1

अब Quizwiz के साथ अपने होमवर्क और परीक्षाओं को एस करें!

A patient is unable to fully extend the right knee because of a 20° knee flexion contracture. Which of the following compensations during the swing phase of the left lower extremity is expected? 1. Hiking of the hip on the left 2. Plantar flexion of the left foot 3. Lateral trunk lean to the left 4. Dropping of the pelvis on the left

1 1. A knee flexion contracture on the stance limb would make it more difficult to clear the opposite leg during midswing. Hip hiking on the left may be performed to attempt to "shorten" the swing leg. 2. Plantar flexion of the swing leg (left) would effectively lengthen the limb, causing further difficulty in clearing the limb. 3. Lateral trunk lean is seen toward the stance side, not the swing side, to reduce abduction demand. 4. Dropping of the pelvis to the left would effectively lengthen the swing leg, causing further difficulty in clearing the limb. Author: O'Sullivan SB, Schmitz TJ, Fulk GDTitle: Physical RehabilitationEdition: 6Publisher: F.A. DavisYear: 2014Pages: 268

A physical therapist strokes the plantar surface of an adult patient's foot with the handle of a reflex hammer. Which of the following responses indicates an upper motor neuron lesion? 1. Extension of the great toe 2. Dorsiflexion of the ankle 3. Flexion of the great toe 4. Withdrawal response

1 1. In a positive reaction, which indicates presence of an upper motor neuron lesion, the great toe extends while the other toes plantar flex and splay. 2. Dorsiflexion of the ankle is not an expected reaction from the stimulus. 3. In a negative reaction, which indicates no abnormality, the toes either do not move at all or flex. 4. The withdrawal response is not an expected reaction from the stimulus. Author: O'Sullivan SB, Schmitz TJ, Fulk GDTitle: Physical RehabilitationEdition: 6Publisher: F.A. DavisYear: 2014Pages: 175

An active 75-year-old patient is admitted to the hospital following a fall at home. All workup is negative and comorbidities are limited to osteoarthritis, cataracts, and hypertension. Which of the following statements is the MOST accurate prognosis? 1. Patient should return to the previous level of function within 1 week. 2. Patient will be independent with a walker on all surfaces in 3 weeks. 3. Patient will need to use a wheelchair at home to avoid falls. 4. Patient should be transferred to a skilled nursing facility for safety.

1 1. A prognosis is the predicted optimal level of improvement in function reached in a certain time period. For some patients, a prognosis may be difficult to establish initially because of complex new problems. In these patients, the prognosis may be established after some treatment and evidence of improvement has occurred. For other patients, like the one in the question, the issues are not complex. The patient has not developed a new illness and will most likely wish to go home. The patient has common illnesses of older persons and will benefit by gait and balance training while in the hospital, with the anticipation of going home to resume normal activities. 2. There is no mention of a gait dysfunction in the question; therefore, assuming the patient needs a walker is inappropriate. 3. The patient should be given an opportunity to ambulate safely before turning to a wheelchair. 4. The patient's workup is negative, the patient was active before admission and the comorbidities are clearly identified. With gait and balance retraining while in the hospital, the patient should be able to resume normal activity. Author: O'Sullivan SB, Schmitz TJ, Fulk GDTitle: Physical RehabilitationEdition: 6Publisher: F.A. DavisYear: 2014Pages: 9

A patient reports left arm pain and has decreased grip strength, decreased triceps reflex, and decreased sensation in the left ring finger (4th digit) and little finger (5th digit). The symptoms are aggravated by turning the head to the left and looking upward. The patient MOSTlikely has which of the following conditions? 1. Cervical nerve root compression 2. Cervical myelopathy 3. Brachial plexus lesion 4. Ulnar nerve lesion

1 1. Cervical radiculopathy (injury to the nerve root) presents with unilateral motor and sensory symptoms into the upper limb, with muscle weakness in the myotome, sensory alteration in the dermatome, and reflex hypoactivity. 2. Cervical myelopathy (injury to the spinal cord itself) presents with spastic weakness, paresthesia, and possible incoordination in one or both lower limbs, and well as sphincter dysfunction. 3. Brachial plexus symptoms are more localized to the shoulder and neck. Cervical positions do not change brachial plexus symptoms. Weakness in the myotome and sensory alteration in the dermatome are common. 4. Peripheral nerve lesions typically do not present with pain. Weakness and sensory symptoms are specific to the nerve involved. Author: Magee DJTitle: Orthopedic Physical AssessmentEdition: 6Publisher: Elsevier SaundersYear: 2014Pages: 160

A 5-year-old child diagnosed with Down syndrome is referred for therapy to facilitate motor development skills. Which of the following activities should the child AVOID? 1. Forward somersaults 2. Cutting paper with scissors 3. Jumping rope 4. Playing catch with a tennis ball

1 1. Children with trisomy 21 frequently have atlantoaxial instability. Rolling and other tumbling activities must be avoided. 2. Blunt scissors should be used to help develop eye-hand coordination and fine motor skills. 3. Gross motor exercises at a level appropriate to the child's ability should be encouraged to facilitate development of gross motor skills. 4. Playing catch should be encouraged to facilitate development of eye-hand coordination and motor skill development. Author: Umphred DA, Lazaro RT, Roller ML, Burton GUTitle: Umphred's Neurological RehabilitationEdition: 6Publisher: Elsevier MosbyYear: 2013Pages: 54, 348-349

A patient has aspiration precautions. Which of the following factors is MOST likely to affect the patient's condition? 1. Liquids are aspirated more easily than solids. 2. Solids are aspirated more easily than liquids. 3. Cold food is easier to swallow than warm food. 4. Hyperextension of the neck facilitates swallowing.

1 1. Dysphagia can lead to aspiration. Dysphagia can be assessed at bedside. Aspiration is more likely to occur with thin liquids. Therefore, treatment is to thicken the liquids or use thicker solutions and then progress to thinner liquids as the patient's swallowing function improves. (McCance, p. 1428) 2. Aspiration is more likely to occur with thin liquids (McCance, p. 1428). 3. Moist, warm food is more easily swallowed (McCance, p. 1428). 4. To facilitate swallowing, posture should be aligned with the chin tucked (Cifu, p. 66). Author: McCance KL, Huether SE, Brashers VL, Rote NSTitle: Pathophysiology: The Biologic Basis for Disease in Adults and ChildrenEdition: 7Publisher: Elsevier MosbyYear: 2014Pages: 1428 Author: Cifu DXTitle: Braddom's Physical Medicine and RehabilitationEdition: 5Publisher: ElsevierYear: 2016Pages: 66

A physical therapist is preparing to change a dressing on a patient who has a large, draining wound. To prevent infection and contamination, the therapist should FIRST: 1. perform hand washing. 2. don a mask. 3. don a sterile gown. 4. wear sterile gloves.

1 1. Hands should be washed before and after donning protective equipment. 2. The therapist should wash hands before donning a mask. 3. The therapist should wash hands and don a mask before donning a gown. 4. Sterile gloves should be donned last. Author: Fairchild SLTitle: Pierson and Fairchild's Principles & Techniques of Patient CareEdition: 5Publisher: Elsevier SaundersYear: 2013Pages: 28 (box 2-2), 36 (box 2-3)

A patient who gave birth 2 weeks ago with a vaginal delivery is referred to physical therapy for a strengthening program. Which of the following muscles is LEAST likely to require strengthening intervention? 1. Hip flexors 2. Trunk flexors 3. Hip extensors 4. Pelvic floor muscles

1 1. Hip flexors are often short due to the lordotic posture of pregnant mothers. Hip flexors would not need strengthening, but lengthening. 2. Trunk flexors are often weak postdelivery since they are put in a lengthened position; strengthening of this muscle group is indicated. 3. Strengthening of the buttock muscles is a typical exercise in postpartum physical therapy care. 4. Pelvic floor muscles should be strengthened, and strengthening can be started within the first 24 hours after delivery to restore tone. Author: Dutton MTitle: Dutton's Orthopaedic Examination, Evaluation, and InterventionEdition: 4Publisher: McGraw-HillYear: 2017Pages: 1589

A physical therapist plans to perform a home assessment for a patient who had a T10 complete spinal cord transection 6 months ago. Which of the following information would be MOST critical to obtain prior to the home visit? 1. Patient's present level of function as judged by the patient and all involved disciplines 2. Patient's previous level of function as judged by the patient and all involved disciplines 3. Results of general health outcome measures 4. Results of disease-specific outcome measures

1 1. It is critical that all stakeholders, including the patient, provide detailed information regarding the patient's current functional level so that the home assessment can be placed within the appropriate context. 2. Information about the patient's previous level of function would be less critical, relative to the patient's current condition and circumstance, for making preparations for the home assessment. 3. Results of general health outcome measures could not provide the necessary preparatory information for the home assessment because they would not provide enough details about the patient's specific condition. 4. Results of disease-specific outcome measures could not provide the necessary preparatory information for the home assessment because they would not provide enough details about the patient's specific condition. Author: O'Sullivan SB, Schmitz TJ, Fulk GDTitle: Physical RehabilitationEdition: 6Publisher: F.A. DavisYear: 2014Pages: 344-345

Which of the following signs and symptoms is MOST likely to be present in an individual with a herniated nucleus pulposus at the L4-L5 level? 1. Pain in the dorsal surface of foot and decreased strength in the extensor hallucis longus 2. Pain in the plantar surface of the foot and decreased ankle inversion strength 3. Pain in the medial aspect of the ankle and decreased strength of the plantar flexors 4. Pain in the lateral aspect of the ankle and decreased ankle eversion strength

1 1. L5 provides sensation to the dorsal area of the foot and innervates the extensor hallucis longus. Because of the way the nerve roots exit L4-L5, a herniated nucleus pulposus at this level usually affects the L5 nerve root rather than L4. 2. S1 provides sensation to the plantar surface of the foot; L4 innervates the ankle invertor (tibialis anterior). 3. L4 provides sensation to the medial aspect of the ankle; S1-S2 innervates the plantar flexors (gastrocnemius and soleus). Because of the way the nerve roots exit L4-L5, a herniated nucleus pulposus at this level usually affects the L5 nerve root rather than L4. 4. S1 provides sensation to the lateral aspect of the foot and ankle and also innervates the muscles that perform eversion (fibularis [peroneus] longus and brevis). Author: Dutton MTitle: Dutton's Orthopaedic Examination, Evaluation, and InterventionEdition: 4Publisher: McGraw-HillYear: 2017Pages: 1494-1495

In order to unlock the knee from terminal extension in a closed kinetic chain, which of the following must occur? 1. Lateral (external) rotation of the femur on the tibia 2. Superior glide of the patella on the femur 3. Lateral (external) rotation of the tibia on the femur 4. Long axis distraction at the knee joint

1 1. Lateral (external) rotation of the femur on the tibia allows the tibiofemoral joint to flex in the weight-bearing position (p. 324). 2. The main function of the patella is to increase the mechanical advantage of the quadriceps and protect the knee joint, which are not primary concerns in this motion. Further, the patella is pulled downward (not superiorly) as the knee flexes. (pp. 324, 326) 3. Evidence does not support the idea of lateral (external) rotation of the tibia on the femur during the initial stages of flexion in the weight-bearing position. The tibia would be able to rotate in an open kinetic chain, but not in a closed kinetic chain (p. 324). 4. Evidence does not support long-axis distraction as a significant component of initial tibiofemoral joint flexion in the weight-bearing position. Author: Lippert LSTitle: Clinical Kinesiology and AnatomyEdition: 6Publisher: F.A. DavisYear: 2017Pages: 324, 326

A patient's exercise program has progressed well following a rotator cuff repair 4 months ago. The patient is about to return to work as a librarian. Which of the following discharge criteria is MOST important for this patient? 1. The patient will be able to lift 5 lb (2.3 kg), 20 times, above shoulder level. 2. The patient will have pain-free active range of motion below shoulder level. 3. The patient will have pain-free active range of motion between 90° to 150° of shoulder flexion. 4. The patient will be able to lift 15 lb (6.8 kg), 5 times, from floor to waist.

1 1. Lifting 5 lb (2.3 kg), 20 times, above shoulder level is a functional goal that should be reached at this stage in the rehabilitation before discharging the patient. In addition, this goal is specific to the patient who is a librarian and will have to lift books above shoulder level throughout the day. 2. Pain-free active range of motion below shoulder level should be reached before the functional phase of rehabilitation in patients who have had a rotator cuff repair. At this point the patient should have 90% to 100% of range of motion restored. 3. Pain-free active range of motion between 90° to 150° of shoulder flexion should be reached before the functional phase of rehabilitation in patients who have had a rotator cuff repair. At this point the patient should have 90% to 100% of range of motion restored. 4. Lifting 15 lb (6.8 kg), 5 times, from floor to waist is not specific to the patient, who is returning to work as a librarian and will not be routinely required to lift objects from floor to waist. Author: Dutton MTitle: Dutton's Orthopaedic Examination, Evaluation, and InterventionEdition: 4Publisher: McGraw-HillYear: 2017Pages: 658

A patient underwent surgical decompression and repair of a large rotator cuff tear 2 weeks ago. Which of the following interventions is LEAST appropriate at this time? 1. Maximal elbow isometric exercises 2. Active elbow flexion and extension through full range 3. Pendulum exercise 4. Passive shoulder flexion to 90°

1 1. Maximal exercises are not indicated at this stage of recovery at the elbow. They are indicated at week 4. 2. Active range motion at the hand, wrist, and elbow is part of the rehabilitation protocol in the early stage of healing. 3. Pendulum exercises are a form of passive range of motion permitted at the shoulder in the early stage of healing. 4. Limited passive range of motion of the shoulder is permitted during the early stage of healing. Author: Reider BC, Davies GJ, Provencher MTTitle: Orthopaedic Rehabilitation of the Athlete: Getting Back in the GameEdition: 1Publisher: Elsevier SaundersYear: 2015Pages: 232-233

A patient has recurrent lateral patellar subluxations. While testing the patient's patellar mobility, the physical therapist notes that the apex of the patella rotates medially during a passive medial patellar glide. Which of the following structures around the patella are tight? 1. Superolateral 2. Superomedial 3. Inferolateral 4. Inferomedial

1 1. Medial glide would stress lateral structures. The apex (inferior pole) of the patella is moving, however the base of the patella (superior) is not, causing patellar rotation. Therefore, superolateral structures are tight, holding back this portion of the patella. 2. With a medial glide, the superomedial structures would be put on slack. 3. Medial glide would stress lateral structures. However, with the apex (inferior pole) of the patella allowed to move (causing the rotation observed), the inferolateral structures are not restricted. 4. With a medial glide, the inferomedial structures would be put on slack. Author: Dutton MTitle: Dutton's Orthopaedic Examination, Evaluation, and InterventionEdition: 4Publisher: McGraw-HillYear: 2017Pages: 981

In which of the following conditions is a nerve conduction velocity test MOST appropriate? 1. Carpal tunnel syndrome 2. Cerebrovascular accident 3. Myotonia 4. Duchenne muscular dystrophy

1 1. Nerve conduction velocity testing is most useful in the evaluation of peripheral nerve or lower motor neuron status. Carpal tunnel syndrome is the only one of the conditions listed that directly involves a peripheral nerve (pp. 1008-1009). 2. Nerve conduction velocity testing is most useful in the evaluation of peripheral nerve or lower motor neuron status. A cerebrovascular accident is a central nervous system disorder not a peripheral nerve disorder (p. 711). 3. With myopathic disorders, motor and sensory nerve conductions are generally normal unless neural tissue is also affected (p. 1014). 4. Nerve conduction velocity testing is most useful in the evaluation of peripheral nerve or lower motor neuron status. Duchenne muscular dystrophy is primarily a muscle disorder (p. 554). Author: Umphred DA, Lazaro RT, Roller ML, Burton GUTitle: Umphred's Neurological RehabilitationEdition: 6Publisher: Elsevier MosbyYear: 2013Pages: 98, 554, 711, 1008-1009, 1014

While a patient is receiving gait training in the use of crutches, the BEST hand placement for the physical therapist to use on the patient is: 1. one hand on the gait belt and one hand on the patient's shoulder. 2. both hands on the gait belt. 3. one hand on the gait belt and one hand on the patient's arm. 4. both hands on the patient's hips.

1 1. Placing one hand on the gait belt and one hand on the patient's shoulder is the correct technique to best facilitate ambulation activity with crutches and provide appropriate guarding. 2. Using both hands on the gait belt reduces the ability to provide tactile inputs to balance and posture during gait. 3. Grasping the arm will retard normal gait movement with the crutches. 4. Placing both hands on the hips will not allow safe control of the patient and will interfere with the therapist's balance if the patient falls forward. Author: Minor MAD, Minor SDTitle: Patient Care SkillsEdition: 7Publisher: PearsonYear: 2014Pages: 347 Author: Fairchild SLTitle: Pierson and Fairchild's Principles & Techniques of Patient CareEdition: 5Publisher: Elsevier SaundersYear: 2013Pages: 224

A physical therapist made two consecutive measurements of right shoulder flexion range of motion with a goniometer. The first measurement was 0°-150°, and the second was 0°-130°. No treatment occurred between measurements, and the remainder of the clinical examination findings were the same at both measurements. The therapist should conclude that the difference in the range of motion is due to poor: 1. intrarater reliability. 2. interrater reliability. 3. content validity of goniometric measurements. 4. criterion-related validity of goniometric measurements.

1 1. Reliability is defined as the amount of consistency between measurements. Intrarater reliability is defined as the consistency of measurements made by the same examiner on different occasions. 2. Reliability is defined as the amount of consistency between measurements. Interrater reliability is defined as the consistency of measurements between two different examiners. 3. Validity is defined as the ability of a measurement tool to test what it is suppose to test. The goniometer is a valid tool for testing range of motion. Content validity is determined by deciding whether the instrument measures and represents the content or the variable in question. 4. Criterion-related validity justifies the validity of an instrument by comparing it to a gold standard of measurement. In the scenario, the comparison is between two instances of the same type of measurement. Author: Portney LG, Watkins MPTitle: Foundations of Clinical Research: Applications to PracticeEdition: 3Publisher: F.A. DavisYear: 2015Pages: 87-88, 101-103

A patient reports a deep, localized pain in the low back and posterior hip region with intermittent tingling that radiates along the posterior thigh. The pain increases when the hip is passively moved into medial (internal) rotation. This finding is MOST likely related to: 1. piriformis syndrome. 2. tension on the posterior joint capsule of the hip. 3. trochanteric bursitis. 4. a sacroiliac joint dysfunction.

1 1. Symptoms described in the scenario are consistent with piriformis syndrome. 2. Tension on the posterior joint capsule of the hip would result in limited motion but not the symptoms described. 3. Trochanteric bursitis produces pain laterally, over the greater trochanter, which is not consistent with the symptoms described in the scenario. 4. Although sacroiliac joint dysfunction can produce back and leg pain, the description is more consistent with piriformis syndrome. Author: Dutton MTitle: Dutton's Orthopaedic Examination, Evaluation, and InterventionEdition: 4Publisher: McGraw-HillYear: 2017Pages: 249-250, 950

Which of the following muscles MUSTbecome elongated for a patient to transition from the posture shown in photograph A to the posture shown in photograph B? 1. Sternocleidomastoids 2. Upper thoracic extensors 3. Lower and middle trapezii 4. Rhomboids major and minor slouched forward head posture

1 1. The sternocleidomastoids are correct because the neck must extend on the thorax and the head must forwardly rotate on the neck to move from A to B. This is opposite the anatomical action of sternocleidomastoids and would require an increase in the distance between their attachments. 2. The upper thoracic extensors are incorrect because moving from A to B would decrease the distance between their attachments, thereby slackening the muscles. These muscles must contract to move from A to B. 3. The lower and middle trapezii are incorrect because moving from A to B would decrease the distance between their attachments, thereby slackening the muscles. These muscles may contract to move from A to B but may not contract at all since the arms are not lifted upward. 4. The rhomboids major and minor are incorrect because they move the shoulder girdles into retraction and downward rotation. This would decrease the distance between their attachments, thereby slackening the muscles. These muscles must contract to move from A to B. Author: Dutton MTitle: Dutton's Orthopaedic Examination, Evaluation, and InterventionEdition: 4Publisher: McGraw-HillYear: 2017Pages: 1311-1312

When a stationary exercise bicycle is used to emphasize strengthening a weak hamstring muscle, which modification to the equipment should be made? 1. Install toe clips on the pedals. 2. Increase the flywheel resistance. 3. Raise the handlebars. 4. Tilt the seat forward.

1 1. Toe clips can limit ankle motion, thereby allowing for increased knee flexion force to facilitate hamstring strengthening (Houglum, pp. 883, 941). 2. Increasing the flywheel resistance does not isolate the hamstring muscles. 3. Raising the handlebars of a bike brings the patient more upright for increased trunk control and balance challenge, not hamstring strengthening (Kisner). 4. Tilting the seat forward on a bike extends the hips and, therefore, places increased load on the trunk, not the hamstrings (Kisner; Houglum, p. 941). Author: Houglum PTitle: Therapeutic Exercise for Musculoskeletal InjuriesEdition: 4Publisher: Human KineticsYear: 2016Pages: 883, 941 Author: Kisner C, Colby LATitle: Therapeutic Exercise: Foundations and TechniquesEdition: 6Publisher: F.A. DavisYear: 2012Pages: 230

A patient who sustained an ulnar collateral ligament sprain of the thumb (1st digit) 1 week ago is referred for physical therapy. Which of the following actions is MOST appropriate when continuous ultrasound is being administered? 1. Immerse the wrist and the hand in water. 2. Apply 10% hydrocortisone to the skin. 3. Apply mineral oil between the skin and the sound head. 4. Utilize a stationary technique in direct contact with the skin.

1 1. Ultrasound consists of acoustic waves that travel poorly through air, but pass readily through water. Because the hand and wrist are small anatomical areas with an irregular surface, immersion in water is an effective means of delivery of ultrasound. 2. Application of 10% hydrocortisone to the skin would not ensure optimal delivery of ultrasound to an irregular surface area of the body. 3. Mineral oil is a poor conductor of sound waves. Oil could also result in damage to the sound head due to overheating because the sound waves are blocked. 4. Holding the ultrasound head stationary results in delivery of sound waves that are too intense. The concentrated energy can cause periosteal pain. Author: Hayes KW, Hall KDTitle: Manual for Physical AgentsEdition: 6Publisher: PearsonYear: 2012Pages: 49

A patient who is performing a vigorous treadmill test is MOST likely to have an immediate increase in which of the following physiologic responses? 1. Respiration rate to raise blood pH levels 2. Respiration rate to lower blood pH levels 3. Rate of excretion of hydrogen ions by the kidneys to raise blood pH levels 4. Rate of excretion of hydrogen ions by the kidneys to lower blood pH levels

1 1. Vigorous exercise produces lactic acid, which would lower the pH of blood unless compensatory mechanisms are in place. Increasing the rate of respiration reduces the alveolar partial pressure of arterial carbon dioxide (PaCO2), resulting in more alkalinity in the blood to prevent an imbalance. (p. 201) 2. Lactic acid pushes the blood ion concentration in the direction of becoming more acidic. The increase in respiration rate helps compensate by increasing blood alkalinity (higher pH). (p. 201) 3. As opposed to the respiratory system, the renal system is involved in more long-term acid-base balance maintenance. The respiratory system plays more of a role in acute responses. (p. 203) 4. As opposed to the respiratory system, the renal system is involved in more long-term acid-base balance maintenance. The respiratory system plays more of a role in acute responses. Lactic acid buildup during exercise leads to a lower pH. The kidneys would work to raise the pH back toward normal. (p. 203) Author: Kenney WL, Wilmore JH, Costill DLTitle: Physiology of Sport and ExerciseEdition: 5Publisher: Human KineticsYear: 2012Pages: 201, 203

The patient in the photograph is unable to tolerate passive lower extremity elevation beyond what is shown. The patient's lower extremity limitation is MOST likely caused by which of the following conditions? 1. Passive insufficiency of the hamstring muscles 2. Capsular tightness in the hip joint 3. Active insufficiency of the hamstrings 4. Tightness in the hip flexor muscles slr with PT holding leg up

1 1. With the straight leg raise, the hamstrings are stretched across both joints at the same time. Because no limitation is noted in the hip or knee joints, the limitation would be due to the hamstrings. (p. 918) 2. The test shown in the photograph is used to evaluate hamstring length. Capsular tightness of the hip would be better assessed with hip joint passive range of motion and end-feels. (p. 903) 3. "Active" would mean that the patient is voluntarily contracting the hamstrings, and the patient is not doing so. 4. The test shown in the photograph is used to evaluate hamstring length. Hip flexor tightness would be better assessed with a Thomas test or passive hip extension range of motion. (p. 916) Author: Dutton MTitle: Dutton's Orthopaedic Examination, Evaluation, and InterventionEdition: 4Publisher: McGraw-HillYear: 2017Pages: 903, 916, 918

A 6-year-old boy presents to physical therapy with right anterior groin pain that sometimes radiates to the right anterior thigh and knee. The patient says, "My leg hurts after I play kickball" but cannot describe a specific incident of trauma. Which of the following information from the patient's chart is MOST important in determining the appropriate diagnosis? 1. Complete blood count 2. Radiograph of the right hip 3. List of prior injuries 4. List of patient's medications

2 1. A complete blood count would be useful if the boy reported more systemic symptoms. 2. A male child of this age is at hip risk for Legg-Calvé-Perthes disease, which frequently presents as knee/thigh pain. A radiograph of the right anterior hip is imperative to rule out this problem. 3. Although a list of prior injuries is helpful in determining a diagnosis, it would not give information about the current status of the hip joint. 4. A medications list would not be most helpful with this case. Author: Palisano RJ, Orlin MN, Schreiber JTitle: Campbell's Physical Therapy for ChildrenEdition: 5Publisher: ElsevierYear: 2017Pages: 319-320

Which of the following hernias is MOST likely to cause shoulder pain? 1. Femoral 2. Hiatal 3. Inguinal 4. Umbilical

2 1. A hiatal hernia would most likely be associated with shoulder pain. A femoral hernia will cause pain in the lateral pelvic wall (Goodman, Pathology, pp. 898-900; Goodman, Differential Diagnosis, pp. 615-615). 2. A hiatal hernia would most likely be associated with shoulder pain. A femoral hernia will cause pain in the lateral pelvic wall. An inguinal hernia will cause groin pain, and an umbilical hernia would most likely cause pain around the umbilical ring in the mid to lower abdomen (Goodman, Pathology, pp. 868, 898-900; Goodman, Differential Diagnosis, pp. 615-616). 3. A hiatal hernia would most likely be associated with shoulder pain. An inguinal hernia will cause groin pain (Goodman, Pathology, pp. 898-900; Goodman, Differential Diagnosis, pp. 615-616). 4. A hiatal hernia would most likely be associated with shoulder pain. An umbilical hernia would most likely cause pain around the umbilical ring in the mid to lower abdomen. (Goodman, Pathology, pp. 898-900; Goodman, Differential Diagnosis, pp. 615-616). Author: Goodman CC, Snyder TEKTitle: Differential Diagnosis for Physical Therapists: Screening for ReferralEdition: 5Publisher: Elsevier SaundersYear: 2013Pages: 615-616 Author: Goodman CC, Fuller KSTitle: Pathology: Implications for the Physical TherapistEdition: 4Publisher: Elsevier SaundersYear: 2015Pages: 868, 898-900

Which of the following are appropriate physical therapy interventions for the management of a joint with heterotopic ossification? 1. Aggressive strengthening of the affected tissues 2. Gentle stretching of the affected tissues 3. Continuous ultrasound 4. Superficial application of heat

2 1. Aggressive stretching is not indicated and should be avoided in patients with heterotopic ossification. 2. Maintaining range of motion with gentle stretching is indicated. 3. Maintaining available range of motion, avoiding "vigorous" stretching, and achieving and maintaining "optimal wheelchair positioning" are the recommended therapeutic interventions. Therapeutic modalities such as ultrasound and superficial heat application have not been shown to effectively address heterotropic ossification. 4. Maintaining available range of motion, avoiding "vigorous" stretching, and achieving and maintaining "optimal wheelchair positioning" are the recommended therapeutic interventions. Therapeutic modalities such as ultrasound and superficial heat application have not been shown to effectively address heterotropic ossification. Author: Umphred DA, Lazaro RT, Roller ML, Burton GUTitle: Umphred's Neurological RehabilitationEdition: 6Publisher: Elsevier MosbyYear: 2013Pages: 478

A patient with multiple sclerosis wants to transfer independently. The patient progressed from moderate to minimal assistance for transfers within 3 days; however, no progress has been made in the past 2 weeks. The physical therapist should: 1. discharge the patient without further intervention. 2. provide the patient with a home program and re-evaluate in 1 month. 3. increase the patient's treatments to 5 times/week for 2 weeks. 4. decrease the patient's treatments to 1 time/week until the goal is achieved.

2 1. Because of the course of exacerbation, relapse, and natural improvement in multiple sclerosis, it is important to provide the patient with a home program instead of discharging. 2. Because of the course of exacerbation, relapse, and natural improvement in multiple sclerosis, it is important to provide the patient with a home program instead of discharging. 3. Increasing the patient's frequency of treatment would likely overfatigue the patient and cause little change in status. 4. Treating the patient 1 time/week would be more of a maintenance program that could be provided for with a home exercise program. Author: Umphred DA, Lazaro RT, Roller ML, Burton GUTitle: Umphred's Neurological RehabilitationEdition: 6Publisher: Elsevier MosbyYear: 2013Pages: 592, 595, 599

A physical therapist is examining an older adult who has a history of falls. The therapist notes that the patient stops walking when engaging in conversation, but resumes walking at a brisk pace as soon as the conversation ends. The MOST appropriate interpretation of this behavior is that the patient has: 1. inadequate cardiopulmonary function to control breathing while walking. 2. high attention requirements for performance of the walking task. 3. difficulty dealing with the small balance perturbations associated with speech production. 4. a long-standing habit of interrupting ongoing activity and standing still to talk.

2 1. Breathlessness is more likely a symptom of cardiopulmonary dysfunction associated with walking and talking. If breathing were difficult when walking at a brisk pace, the patient would more likely slow down to have enough breath for both talking and walking. 2. This finding suggests the individual is struggling with the introduction of a secondary task (conversation), which may interfere with performance of a primary task (walking). Individuals who must devote extra attention to walking are unable to perform the two tasks simultaneously. 3. Speech should not typically create balance perturbations. 4. It is more likely that the person stops walking when talking to concentrate on the task of walking, for which talking is a distraction. Author: O'Sullivan SB, Schmitz TJ, Fulk GDTitle: Physical RehabilitationEdition: 6Publisher: F.A. DavisYear: 2014Pages: 236, 561

During initial examination of a patient, a physical therapist notices severe clubbing of the patient's fingernails. The therapist should further investigate for the presence of signs and symptoms associated with which of the following conditions? 1. Skin cancer 2. Renal failure 3. Lung cancer 4. Liver dysfunction

3 1. Nail clubbing is not associated with skin cancer. 2. The nails of patients who have renal failure may appear to have short transverse lines across the nail (Mees lines) or a brownish distal one-third end of the nail (half-and-half nails), but not clubbing. 3. Severe clubbing of the nails is an abnormality associated with lung cancer and chronic hypoxia. 4. Liver dysfunction may result in nails with transverse depressions (Beau lines) or a nail bed that is white and extends two-thirds of the length of the nail (Terry nails). Author: O'Sullivan SB, Schmitz TJ, Fulk GDTitle: Physical RehabilitationEdition: 6Publisher: F.A. DavisYear: 2014Pages: 500

Which of the following factors is MOST likely to increase the effectiveness of an educational presentation on body mechanics for a group of industrial workers? 1. Objectives are made clear during the presentation. 2. Workers feel ready and willing for behavior change. 3. Material is presented in a video format. 4. Material is presented in a lecture format.

2 1. Clear objectives are important, but not as effective as motivation to change. 2. Understanding the process of individualized behavior change is the key to effective patient education. Once the patient is ready, willing, and able to make the recommended changes, instructional material can be introduced. 3. Presentation format is important, but not as effective as motivation to change. 4. Presentation format is important, but not as effective as motivation to change. Author: Davis CM, Musolino GMTitle: Patient Practitioner Interaction: An Experiential Manual for Developing the Art of Health CareEdition: 6Publisher: Slack IncYear: 2016Pages: 269

A physical therapist is working with a patient who has complete loss of vestibular function. As a compensatory strategy, the therapist should work with the patient on exercises that will: 1. facilitate agility. 2. improve gaze stability. 3. improve coordination. 4. increase muscle strength.

2 1. Facilitating agility is not a mechanism for recovery in patients with complete loss of vestibular function. 2. Improving gaze stability is a well-studied mechanism for recovery from loss of vestibular function. 3. Improving coordination is not a mechanism for recovery in patients with complete loss of vestibular function. 4. Increasing muscle strength is not a mechanism for recovery in patients with complete loss of vestibular function. Author: O'Sullivan SB, Schmitz TJ, Fulk GDTitle: Physical RehabilitationEdition: 6Publisher: F.A. DavisYear: 2014Pages: 968, 983

An inpatient is referred to physical therapy after undergoing coronary artery bypass surgery 5 days ago. The patient's medical history includes hypertension, hypercholesterolemia, and type 2 diabetes. Which of the following sets of factors should a physical therapist consider when developing a plan of care? 1. History of smoking, electrocardiographic changes, and parental/family history 2. Premorbid physical activity level, current physical condition, and motivation to exercise 3. Lower extremity muscle strength, waist-to-chest ratio measurement, and endurance on treadmill test 4. Exercise history, daily caloric intake and dietary habits, and job responsibilities

2 1. Genetic factors and family history are not modifiable factors and cannot be addressed by the physical therapist's treatment plan. Electrocardiographic changes are also not addressable by the physical therapist. 2. Physical activity level, current physical condition, and motivation to exercise are modifiable factors. These factors can be addressed in the plan of care. (McArdle) 3. Lower extremity muscle strength, waist-to-chest ratio measurement, and endurance on treadmill test are not relevant to progression of coronary artery disease and do not need to be included specifically in the plan of care. 4. Exercise history, daily caloric intake and dietary habits, and job responsibilities are not addressable by a physical therapist's plan of care. Collecting dietary data is not best because the physical therapist cannot design dietary programs for patients. It is more important to take into account the modifiable factors when developing the plan of care. Author: McArdle WD, Katch FI, Katch VLTitle: Exercise Physiology: Nutrition, Energy, and Human PerformanceEdition: 8Publisher: Wolters Kluwer HealthYear: 2015Pages: 902 Author: Rosendorff CTitle: Essential Cardiology: Principles and PracticeEdition: 3Publisher: SpringerYear: 2013Pages: 405

Which of the following interventions is MOST appropriate for treating complex regional pain syndrome in a patient recovering from traumatic brain injury? 1. Active range of motion, splinting, and whirlpool therapy 2. Active range of motion, functional training, and transcutaneous electrical nerve stimulation 3. Passive range of motion, splinting, and contrast baths 4. Passive range of motion, functional training, and moist hot packs

2 1. Immobilization may exacerbate symptoms. Elevation of the involved limb is preferred, making whirlpool a less desirable modality. (Dutton, p. 855) 2. Active range of motion, functional training, and transcutaneous electrical nerve stimulation are beneficial in the rehabilitation of a patient who is recovering from complex regional pain syndrome (Dutton, p. 855; Sluka, pp. 364-367). Modalities for pain control may be adjuvant, which can include transcutaneous electrical nerve stimulation (Michlovitz, pp. 334, 345). 3. Early treatment for complex regional pain syndrome should emphasize active, not passive, range of motion. Immobilization from splinting may exacerbate symptoms. (Dutton, p. 855) 4. Early treatment for complex regional pain syndrome should emphasize active, not passive, range of motion (Dutton, p. 855; Sluka, pp. 364-367). Author: Sluka KATitle: Mechanisms and Management of Pain for the Physical TherapistEdition: 2Publisher: IASP PressYear: 2016Pages: 364-367 Author: Dutton MTitle: Dutton's Orthopaedic Examination, Evaluation, and InterventionEdition: 4Publisher: McGraw-HillYear: 2017Pages: 273, 854-855 Author: Michlovitz SL, Bellew JW, Nolan TP JrTitle: Modalities for Therapeutic InterventionEdition: 5Publisher: F.A. DavisYear: 2012Pages: 334, 345

When using biofeedback for neuromuscular re-education on a patient with a traumatic brain injury, the physical therapist should FIRST: 1. place the electrodes over the appropriate muscle. 2. determine that the patient is able to understand the concept of feedback. 3. set the biofeedback unit's threshold to the lowest level. 4. use both audio and visual feedback modes.

2 1. Placing the electrodes prior to determining if the patient is mentally alert with intact thought processes would be inappropriate in this case. The patient should demonstrate understanding before proceeding with the set-up. 2. Determination that the patient is mentally alert and has intact thought processes regarding treatment should precede other treatment procedures. 3. Setting the biofeedback to its lowest levels is not the initial priority; the patient's ability to comprehend the treatment must first be ascertained. 4. Use of both audio and visual feedback modes may be appropriate, but only after determining the patient understands the treatment. If the patient does not understand the intervention, it should not be used. Author: Umphred DA, Lazaro RT, Roller ML, Burton GUTitle: Umphred's Neurological RehabilitationEdition: 6Publisher: Elsevier MosbyYear: 2013Pages: 779, 1025

A physical therapist is asked to analyze physical therapy notes in the medical record to assess peer performance. This process is MOST likely to be performed during which of the following activities? 1. Productivity analysis 2. Quality assurance 3. Utilization review 4. Accreditation review

2 1. Productivity analysis is a measure of financial efficiency of labor costs; reviewing physical therapy notes reflects quality, not efficiency (Page, pp. 100-101). 2. Peer review of patient care notes is a measure of quality (Osborne, p. 205). 3. Utilization review is an external review of the necessity of patient care, not a review of the quality of patient care (Pagliarulo, p. 136). 4. The accreditation process is a voluntary process of quality measurement against an external set of standards (Page, p. 260). Author: Page CGTitle: Management in Physical Therapy PracticesEdition: 2Publisher: F.A. DavisYear: 2015Pages: 100-101, 260 Author: Pagliarulo MATitle: Introduction to Physical TherapyEdition: 5Publisher: ElsevierYear: 2016Pages: 142 Author: Osborne JATitle: Documentation for Physical Therapy Practice: A Clinical Decision Making ApproachEdition: 1Publisher: Jones and BartlettYear: 2016Pages: 205

Which of the following conditions is associated with the GREATEST risk for developing stress urinary incontinence? 1. Multiple sclerosis 2. Post multiparous delivery 3. Post herpes zoster 4. Diabetes mellitus

2 1. Seventy percent to 90% of people with multiple sclerosis develop bladder dysfunction or a neurogenic bladder. This disorder is caused by damage to the nerves that control the urinary tract. Development of stress incontinence is typically related to insufficient strength of pelvic floor musculature. 2. Stress incontinence is essentially due to insufficient strength of the pelvic floor musculature. Pelvic floor dysfunction is most often caused by a weakening of the pelvic floor structures as a result of childbirth or overstretching of the muscles. 3. Herpes zoster is an infection that may cause urinary incontinence as a result of an upper motor neuron (spinal cord) lesion. It causes neurogenic bladder, not stress incontinence. 4. Diabetes mellitus is a common cause of neurogenic bladder dysfunction causing overflow incontinence. Also called cystopathy, the neurogenic bladder is considered a form of autonomic neuropathy. It begins with selective damage to autonomic afferent nerves, leaving motor function intact but impairing the sensation of bladder fullness and, therefore, resulting in decreased urinary frequency. As this neuropathy progresses, autonomic efferent nerves become involved, leading to incomplete bladder emptying, urinary dribbling, and overflow incontinence. This process does not lead to stress incontinence. Author: Moini JTitle: Introduction to Pathology for the Physical Therapist AssistantEdition: 1Publisher: Jones and BartlettYear: 2013Pages: 436

A patient reports difficulty in raising the right arm. During examination, manual muscle testing of shoulder abduction reveals strength of Good (4/5) without pain. Which of the following is the MOST likely cause of these findings? 1. Deltoid tendinitis 2. Suprascapular nerve damage 3. Subacromial bursitis 4. Supraspinatus tendinitis

2 1. Tendinitis results in painful weakness since it affects a contractile structure (p. 650). 2. Weakness without pain implicates a nerve injury. Of the options, suprascapular nerve damage is the only nerve injury. (pp. 78-79) 3. Subacromial bursitis is likely to be painful with potential weakness, but it may be associated with normal strength. Bursitis may be painful with passive mobility testing. (p. 684) 4. Tendinitis results in painful weakness since it affects a contractile structure (p. 650). Author: Dutton MTitle: Dutton's Orthopaedic Examination, Evaluation, and InterventionEdition: 4Publisher: McGraw-HillYear: 2017Pages: 78-79, 650, 684

Which of the following glenohumeral positions is BESTfor maximal stretch of the latissimus dorsi? 1. Extension and lateral (external) rotation 2. Flexion and lateral (external) rotation 3. Extension and medial (internal) rotation 4. Flexion and medial (internal) rotation

2 1. The best position to stretch the latissimus dorsi is flexion and lateral (external) rotation, not extension. 2. The latissimus dorsi is a glenohumeral extensor and medial (internal) rotator. To stretch, the motions are opposite: flexion and lateral (external) rotation. 3. The best position to stretch the latissimus dorsi is flexion and lateral (external) rotation, not extension and medial (internal) rotation. 4. The best position to stretch the latissimus dorsi is flexion and lateral (external) rotation, not medial (internal) rotation. Author: Moore KL, Dalley AF, Agur AMRTitle: Clinically Oriented AnatomyEdition: 7Publisher: Lippincott, Williams & WilkinsYear: 2014Pages: 700

A patient reports pain over the sacroiliac joint and buttock on the left side. The patient's pain is reproduced with an ipsilateral active straight leg raise, sacroiliac compression test, and palpation of the left sacrotuberous ligament. In standing, the right posterior superior iliac spine is higher than the left, and the left anterior superior iliac spine is higher than the right. Interventions should MOST likely include unilateral isometric contraction of the left hip: 1. extensors. 2. flexors. 3. abductors. 4. adductors.

2 1. The clinical signs are indicative of posterior rotation of the left innominate. Isometric contraction of the ipsilateral hip extensors would not correct the problem. 2. The clinical signs are indicative of posterior rotation of the left innominate. Isometric contraction of the ipsilateral hip flexors is a muscle energy technique to correct the problem. 3. The clinical signs are indicative of posterior rotation of the left innominate. Isometric contraction of the ipsilateral hip abductors would not correct the problem. 4. The clinical signs are indicative of posterior rotation of the left innominate. Isometric contraction of the ipsilateral hip adductors would not correct the problem. Author: Dutton MTitle: Dutton's Orthopaedic Examination, Evaluation, and InterventionEdition: 4Publisher: McGraw-HillYear: 2017Pages: 1561

When evaluating the lower extremity muscle strength of a patient, the physical therapist positions the patient prone with the knee flexed. The therapist asks the patient to point the toes upward toward the ceiling. The patient completes the motion, but inverts the foot slightly. This observation indicates: 1. tightness of the fibularis (peroneal) muscles. 2. substitution by the soleus. 3. substitution by the tibialis posterior. 4. tightness of the tibialis anterior.

3 1. Fibularis (peroneal) muscle tightness can lead to foot eversion, not inversion (p. 631). 2. Substitution of the soleus is neutral with respect to foot inversion and eversion (p. 633). 3. Substitution of the tibialis posterior may lead to an inverted foot (pp. 634-635). 4. Tightness of the tibialis anterior may limit the degree of plantar flexion achieved but would not influence foot inversion or eversion (p. 630). Author: Neumann DATitle: Kinesiology of the Musculoskeletal System: Foundations for RehabilitationEdition: 3Publisher: ElsevierYear: 2017Pages: 630-631, 633-635

A patient reports a 2-week history of left upper abdominal quadrant pain, left flank pain, and mid-back pain as a result of a motor vehicle accident. The patient also reports being fatigued and generally not feeling well. Which of the following differential diagnoses would MOST likely account for the patient's symptoms? 1. Chronic fatigue syndrome 2. Referred pain from the spleen 3. Conversion disorder from emotional distress 4. Acute onset of bladder infection

2 1. The diagnostic criteria for chronic fatigue syndrome are similar to those for fibromyalgia syndrome (FMS) with the hallmark feature being fatigue (versus pain in FMS). This option does not address specifically the left upper abdominal quadrant pain, left flank pain, and mid-back pain. (p. 460) 2. The spleen is located in the upper left abdominal quadrant. Enlargement of the spleen may be associated with the etiology and symptoms noted in the stem. (pp. 214-215) 3. A conversion disorder is defined as a condition that presents as an alteration or loss of a physical function suggestive of a physical disorder, with often an underlying psychological conflict or need. The stem does not describe a loss of function. (p. 145) 4. Lower abdominal pain and a strong urge to urinate are characteristics of a bladder infection. These symptoms are not described in the stem. (p. 628) Author: Goodman CC, Snyder TEKTitle: Differential Diagnosis for Physical Therapists: Screening for ReferralEdition: 5Publisher: Elsevier SaundersYear: 2013Pages: 145, 214-215, 628

A patient has a diagnosis of right C5 radiculopathy. Which of the following arm movements is MOST likely to be weak? 1. Elbow extension 2. Shoulder abduction 3. Wrist extension 4. Cervical spine lateral flexion

2 1. The elbow extensors are innervated by C7, not C5. 2. The shoulder abductors are innervated by C5 and would be expected to be weak. 3. The wrist extensors are innervated by C6, not C5. 4. The cervical spine lateral flexors are innerved by the C3-C4nerve roots and the spinal accessory nerve (CN XI), not C5. Author: O'Sullivan SB, Schmitz TJ, Fulk GDTitle: Physical RehabilitationEdition: 6Publisher: F.A. DavisYear: 2014Pages: 6, 154

A physical therapist evaluating the posture of an obese patient notes that the patient has an anterior pelvic tilt. The therapist will MOST likely find weakness in the: 1. iliopsoas and erector spinae. 2. rectus abdominis and gluteus maximus. 3. latissimus dorsi and sartorius. 4. quadratus lumborum and biceps femoris.

2 1. The iliopsoas and erector spinae are placed in a shortened position in an individual who has an anterior pelvic tilt and are likely to be strong, rather than weak (p. 1452). 2. The rectus abdominis and gluteus maximus are placed in a lengthened state in an individual who has an anterior pelvic tilt and are likely to be weak (p. 1452). 3. Both the sartorius and latissimus dorsi muscles will be in a shortened and tight position, not a weakened and elongated position. 4. The quadratus lumborum is a spine stabilizer and more involved in side bending than an anterior or posterior pelvic tilt (p. 1430). Weakness of the biceps femoris would be manifested in the motions of hip extension, knee flexion, and lateral (external) rotation of the tibia (p. 977). Author: Dutton MTitle: Dutton's Orthopaedic Examination, Evaluation, and InterventionEdition: 4Publisher: McGraw-HillYear: 2017Pages: 977, 1430, 1452

A physical therapist is treating a patient with biofeedback to alter the overactivity of the upper trapezius during shoulder flexion. The patient has improved by 50% but continues to inappropriately recruit the upper trapezius muscle. Which of the following courses of action for biofeedback should the therapist take NEXT? 1. Decrease the sensitivity (gain). 2. Increase the sensitivity (gain). 3. Maintain the same sensitivity (gain). 4. Modify the sensitivity (gain) until visible motor contraction is reached.

2 1. The sensitivity should be set higher to be more sensitive or selective to the firing of motor fibers so the patient can respond to inappropriate motor recruitment. 2. The sensitivity or "gain" should be increased as the patient learns to decrease or "train out" muscle recruitment. With increased sensitivity, the patient will work "harder" to not recruit the muscle (inhibition). The patient is attempting to learn how to decrease muscle firing/motor unit activity during voluntary movement. 3. The same sensitivity would not be as effective. The patient has already learned to decrease recruitment of the upper trapezius by 50%; therefore, to continue toward the goal of "completely training out" this recruitment, the sensitivity should be increased. With increased sensitivity, the patient will work "harder" to not recruit the muscle (inhibition). The patient is attempting to learn how to decrease muscle firing/motor unit activity during voluntary movement. 4. Unlike neuromuscular electrical stimulation, biofeedback does not produce a visible muscle contraction. It only helps with recruitment of motor units (contracting muscle) and displays the activity of the muscle being monitored. Furthermore, this patient should decrease, not increase, motor unit recruitment. Therefore, eliciting a visible muscle contraction would not be helpful, as this would increase the activity of the motor units. Author: Hayes KW, Hall KDTitle: Manual for Physical AgentsEdition: 6Publisher: PearsonYear: 2012Pages: 176-177

The joint mobilization technique seen in the illustration is designed to improve hip: 1. external rotation. 2. flexion. 3. internal rotation. 4. adduction. pt supine with right leg resting on PT shoulder. PT is holding onto upper thigh and distracting distally. kind of looks like a scooping motion

2 1. To increase lateral (external) rotation, an anterior glide would be used. The photograph shows an inferior glide, which is used to increase hip flexion. 2. The photograph shows a inferior glide, which is used to increase hip flexion. 3. To increase medial (internal) rotation, a posterior glide would be used. The photograph shows an inferior glide, which used to increase hip flexion. 4. The photograph shows an inferior glide, which is used to increase hip flexion, not adduction. Author: Dutton MTitle: Dutton's Orthopaedic Examination, Evaluation, and InterventionEdition: 4Publisher: McGraw-HillYear: 2017Pages: 954

A patient has completed a 12-week strength and conditioning program. Which of the following measures would be MOST appropriate to assess change in fitness and conditioning from week 1 to week 12? 1. Resting respiration rate before starting exercise 2. Amount of time until the heart rate returns to baseline after exercise 3. Amount of time until the onset of perspiration during exercise 4. Amount of increase in diastolic blood pressure during exercise

2 1. Training does not affect pulmonary ventilation at rest (p. 259). 2. After training, heart rate returns to resting levels much more quickly after exercise, and time to return to baseline is sometimes used as a measure of physical fitness (p. 255). 3. Time until the onset of perspiration is not an easily quantifiable measure and also may vary as a result of other factors, i.e., hydration (p. 381). 4. Following training, blood pressure is reduced at submaximal intensities. The amount of increase in diastolic blood pressure during exercise is not a measure of cardiorespiratory fitness. (pp. 255, 257) Author: Kenney WL, Wilmore JH, Costill DLTitle: Physiology of Sport and ExerciseEdition: 5Publisher: Human KineticsYear: 2012Pages: 255, 257, 259, 381

A 13-year-old male patient who is morbidly obese has a recent onset of left knee pain. His left hip is in slight flexion, adduction, and lateral (external) rotation. Examination shows no medial (internal) hip rotation and abduction, with increasing pain on abduction. Which of the following conditions is MOST likely present? 1. Weakness in the left hip abductor musculature 2. Left slipped capital femoral epiphysis 3. Avascular necrosis of the left femoral head 4. Dislocated left hip

2 1. Weakness in left hip abductor musculature would not account for positioning in hip flexion and lateral (external) rotation, which together are indicative together of slipped capital femoral epiphysis (p. 892). 2. This patient demonstrates the clinical indicators of slipped capital femoral epiphysis, including deficits in hip medial (internal) rotation and abduction (pp. 892, 1580). 3. The clinical indicators avascular necrosis of the femoral head include pain in the groin, proximal thigh, or buttock area that is usually exacerbated by weight-bearing (p. 256), not by hip abduction as in a slipped capital femoral epiphysis (p. 892). 4. Other than demonstrating pain with hip abduction, the patient does not demonstrate the clinical indicators of a dislocated hip, which include severe groin and lateral hip pain and the hip held in abduction (pp. 255, 892). Author: Dutton MTitle: Dutton's Orthopaedic Examination, Evaluation, and InterventionEdition: 4Publisher: McGraw-HillYear: 2017Pages: 255-256, 892, 1580

A physical therapist is preparing to give instructions in the use of a new standing device to the family of a 3-year-old child who has myelomeningocele. Which of the following actions should the therapist perform FIRST? 1. Ask the family what problems they would have with using a standing device. 2. Give the family product literature and a video about the standing device. 3. Ask the family what method would be best for them to learn to use the standing device. 4. Show the family how to place the child into the standing device.

3 1. A coaching strategy is capacity building, nondirective, and solution focused, which is not achieved by focusing on asking about anticipated problems. 2. Literature and video may be an appropriate learning activity, but only after asking a question such as, "Would having a video of doing the strategy be helpful?" 3. A coaching approach supports the family in expanding current abilities and learning new skills. Asking the family what learning method is best for them is consistent with adult learning and is solution focused, which is the premise of this question. 4. Showing the family how to do something may be an appropriate learning activity, but only after they are asked a question such as, "What can I do to make it comfortable for you to try this strategy?" Author: Palisano RJ, Orlin MN, Schreiber JTitle: Campbell's Physical Therapy for ChildrenEdition: 5Publisher: ElsevierYear: 2017Pages: 710-711

Which of the following observations indicates an improvement in a patient with dysdiadochokinesia? 1. Decreased resting tremor 2. Increased one-foot standing balance 3. Increased typing speed 4. Decreased horizontal nystagmus

3 1. A resting tremor is most prominent at rest and may decrease or disappear with voluntary movement. Resting tremor is not a component of dysdiadochokinesia. (p. 220) 2. Single limb support is not related to dysdiadochokinesia (p. 220). 3. Dysdiadochokinesia is characterized by the inability to perform quick, alternating movements. Typing speed is dependent on the ability to place and lift the fingers quickly. (p. 220) 4. Nystagmus is the rhythmic oscillation of the eyes and is not a component of dysdiadochokinesia (p. 212). Author: O'Sullivan SB, Schmitz TJ, Fulk GDTitle: Physical RehabilitationEdition: 6Publisher: F.A. DavisYear: 2014Pages: 212, 220

A patient's electrocardiogram shows a new ST-segment displacement from baseline and a sinus rhythm of 70 bpm. What is the MOST likely diagnosis? 1. Bradycardia 2. Anxiety reaction 3. Acute myocardial infarction 4. Congestive heart failure

3 1. Bradycardia is a heart rate below 60 bpm (pp. 314-315). 2. Anxiety reaction would usually include an increased heart rate (pp. 315-316). 3. Acute myocardial infarction is associated with either ST elevation or ST depression (pp. 285, 331). 4. Congestive heart failure is not usually associated with ST elevation. Author: Hillegass ETitle: Essentials of Cardiopulmonary Physical TherapyEdition: 4Publisher: ElsevierYear: 2017Pages: 285, 314-316, 331

Electromyographic biofeedback recorded from the quadriceps femoris during a unilateral squat provides information regarding which of the following factors related to the muscle? 1. Endurance 2. Length 3. Activity 4. Force

3 1. Electromyographic biofeedback reflects the amplitude of muscle activation. The signal does not contain any information about muscle endurance. 2. Electromyographic feedback is a recording of the voltage changes associated with the activation of skeletal muscle fibers. The signal does not contain any information about muscle length. 3. Electromyographic feedback is a recording of the voltage changes associated with the activation of skeletal muscle fibers. The patient can get auditory and/or visual feedback about the amplitude of muscle activity without intermediate feedback from the therapist. 4. No direct information is provided by electromyographic feedback regarding the force that a muscle produces. The relationship between electromyographic feedback and muscle force holds only under carefully controlled isometric conditions, not for the nonisometric contraction described here. Author: Cameron MHTitle: Physical Agents in Rehabilitation: From Research to PracticeEdition: 4Publisher: Elsevier SaundersYear: 2013Pages: 75

A physical therapist is developing an exercise program for a patient who has upper extremity lymphedema. Which of the following exercises should the patient perform LAST? 1. Elbow flexion 2. Cervical rotation 3. Wrist circumduction 4. Shoulder circumduction

3 1. Exercises are performed from proximal to distal areas (p. 976). Active elbow exercise is indicated but would be performed before exercises for the wrist because the elbow is a proximal segment relative to the wrist (pp. 975-976). 2. The exercise program for lymphedema should begin with total body relaxation. Cervical rotation is an example of this initial exercise. Exercises are prescribed from proximal to distal areas. (p. 976) 3. Exercises are performed from proximal to distal areas (p. 976). Wrist circumduction is performed last because the wrist is the most distal segment (pp. 975-976). 4. Exercises are performed from proximal to distal areas (p. 976). Shoulder circumduction represents proximal clearance (p. 977). Author: Kisner C, Colby LATitle: Therapeutic Exercise: Foundations and TechniquesEdition: 6Publisher: F.A. DavisYear: 2012Pages: 975-977

A patient reports insidious onset of mild low back pain that began 1 week ago without a history of trauma. Which of the following symptoms would MOST suggest immediate referral to a physician? 1. Back pain increases with the lower extremity raised and neck flexed in sitting. 2. Pain centralizes with trunk extension. 3. Percussion over the costovertebral angle causes pain. 4. Back pain increases with pressure applied lateral to the anterior superior iliac spines.

3 1. Neck flexion along with a straight leg raise may indicate irritation to the dura and may indicate a disc lesion. If the pain was very severe, it may warrant a referral, but it is a common sign found with disc lesions and may not warrant an immediate referral. (Cook, p. 302) 2. Centralization of pain with trunk extension would be managed with an extension exercise program, not a referral to a physician (Cook, p. 299). 3. Percussion over the costovertebral angle (Murphy test) is a sign of a kidney disorder. A patient with this sign should be referred, since the problem may not be musculoskeletal. (Goodman) 4. Pressure applied lateral to the anterior superior iliac spines is a common test for sacroiliac pain which would not warrant an immediate referral (Cook, p. 330). Author: Goodman CC, Snyder TEKTitle: Differential Diagnosis for Physical Therapists: Screening for ReferralEdition: 5Publisher: Elsevier SaundersYear: 2013Pages: 215 Author: Cook CE, Hegedus EJTitle: Orthopedic Physical Examination Tests: An Evidence-Based ApproachEdition: 2Publisher: PearsonYear: 2013Pages: 299, 302, 330

A physical therapist is considering an intervention for a patient with lymphedema and severe arterial insufficiency. Which of the following options BEST represents the rationale for a decision concerning use of intermittent compression therapy? 1. Should be used, because it increases peripheral resistance. 2. Should be used, because it moves excess interstitial fluid. 3. Should not be used, because it increases peripheral resistance. 4. Should not be used, because it moves excess interstitial fluid.

3 1. Patients who have arterial insufficiency have increased peripheral resistance, and compression would increase it further. Therefore, this modality should not be used on patients who have arterial insufficiency. 2. Normal lymphatic drainage is generally disrupted in patients who have lymphedema, and intermittent compression would be used to treat the edema by moving interstitial fluid. However, due to the presence of arterial insufficiency, compression should not be used. 3. Patients who have arterial insufficiency have increased peripheral resistance, and compression would increase it further. Therefore, this modality should not be used on patients who have arterial insufficiency. 4. Normal lymphatic drainage is generally disrupted in patients who have lymphedema, and intermittent compression would be used to treat the edema by moving interstitial fluid. However, compression should not be used in patients who have arterial insufficiency, but the reason is that compression would increase peripheral resistance, not because it moves excess interstitial fluid. Author: Cameron MHTitle: Physical Agents in Rehabilitation: From Research to PracticeEdition: 4Publisher: Elsevier SaundersYear: 2013Pages: 390, 395, 398

A patient with relapsing-remitting multiple sclerosis will MOST likely experience which of the following over time? 1. Continuous and steady decline, followed by a plateau 2. Acute worsening, followed by full recovery without additional episodes 3. Acute worsening, followed by improvement and disease stability 4. Continuous and steady decline without improvement

3 1. Primary progressive multiple sclerosis is characterized by a continuous, steady decline with or without occasional plateaus. 2. All forms of multiple sclerosis are characterized by either relapses or steady progression of the disease. 3. Relapsing-remitting multiple sclerosis is characterized by clearly defined acute attacks with full recovery or with residual deficits after recovery. 4. Primary progressive multiple sclerosis is characterized by a continuous, steady decline with or without occasional plateaus. Author: Umphred DA, Lazaro RT, Roller ML, Burton GUTitle: Umphred's Neurological RehabilitationEdition: 6Publisher: Elsevier MosbyYear: 2013Pages: 586

A patient with syndrome of inappropriate antidiuretic hormone secretion (SIADH) would MOST likely have complications involving which of the following systems? 1. Neuromuscular 2. Integumentary 3. Cardiovascular 4. Musculoskeletal

3 1. Syndrome of inappropriate antidiuretic hormone secretion results in fluid volume excess. Fluid loss would be more likely to result in neuromuscular symptoms, such as tetany or tingling. (p. 211) 2. The skin may be warm or cool if edema is present, but there should be no concern with skin integrity in this condition (p. 211). 3. Syndrome of inappropriate antidiuretic hormone results in fluid volume excess, so it may cause hypertension and arrhythmias, which require monitoring as activity levels change. Also the physical therapist may observe distended neck veins or a visible jugular pulse. (pp. 211, 482-483) 4. Syndrome of inappropriate antidiuretic hormone secretion results in fluid volume excess. Fluid loss would be more likely to result in musculoskeletal symptoms, such as weakness. (p. 211) Author: Goodman CC, Fuller KSTitle: Pathology: Implications for the Physical TherapistEdition: 4Publisher: Elsevier SaundersYear: 2015Pages: 211, 482-483

A patient, who has many risk factors for coronary artery disease and is presently not taking any cardiac medications, is interested in beginning an exercise program at a gym to improve cardiac health. The BESTself-assessment of exercise intensity during the exercise sessions of this patient is: 1. change in systolic blood pressure. 2. MET (metabolic equivalent) level. 3. rating of perceived exertion. 4. respiratory rate.

3 1. Systolic blood pressure is a useful measure of exercise intensity, but it is not easily assessed by the patient (Hillegass, p. 558). 2. A MET is a metabolic equivalent of an individual's resting metabolic rate. It varies based upon individual factors and is generally too complex to be a useful self-assessment method. (Kraemer, p. 56) 3. The rating of perceived exertion scale is subjective and is a fairly accurate assessment of ventilatory threshold (Hillegass, pp. 558, 582). 4. Respiratory rate is a useful measure of exercise intensity, but it is not easily assessed by the patient (Hillegass, p. 558). Author: Hillegass ETitle: Essentials of Cardiopulmonary Physical TherapyEdition: 4Publisher: ElsevierYear: 2017Pages: 558, 582 Author: Kraemer WJ, Fleck SJ, Deschenes MRTitle: Exercise Physiology: Integrating Theory and ApplicationEdition: 1Publisher: Lippincott, Williams & WilkinsYear: 2012Pages: 56

A patient with a flaccid upper extremity and downward rotation of the scapula is MOST at risk for which type of glenohumeral joint dislocation? 1. Posterior 2. Anterior 3. Inferior 4. Superior

3 1. The most common type of subluxation is an inferior subluxation. Weakness and the weight of a heavy arm result in downward rotation of the scapula, not a posterior subluxation. 2. The most common type of subluxation is an inferior subluxation. Weakness and the weight of a heavy arm result in downward rotation of the scapula, not an anterior subluxation. 3. The most common type of subluxation is an inferior subluxation. Weakness and the weight of a heavy arm result in downward rotation of the scapula. 4. The most common type of subluxation is an inferior subluxation. Weakness and the weight of a heavy arm result in downward rotation of the scapula, not a superior subluxation. Author: Umphred DA, Lazaro RT, Roller ML, Burton GUTitle: Umphred's Neurological RehabilitationEdition: 6Publisher: Elsevier MosbyYear: 2013Pages: 739

The ankle malalignment shown in the photograph is MOST correctly described by which of the following positions of the calcaneus, forefoot, and subtalar joints, respectively? 1. Valgum, abduction, and inversion 2. Varum, adduction, and eversion 3. Valgum, abduction, and eversion 4. Varum, abduction, and inversion pes planus

3 1. The picture demonstrates a pes planus with foot pronation. The subtalar joint's main components with this are eversion and abduction (Neumann, p. 609). The midtarsal joint (the transverse tarsal joints) also everts (Neumann, p. 614). In stance, the calcaneus is excessively everted from midline, which is a valgus position (Neumann, p. 619). With pronation there is also associated abduction of the forefoot (Magee, p. 896). Subtalar inversion is incorrect. 2. The picture demonstrates a pes planus with foot pronation. The subtalar joint's main components with this are eversion and abduction (Neumann, p. 609). The midtarsal joint (the transverse tarsal joints) also everts (Neumann, p. 614). In stance, the calcaneus is excessively everted from midline, which is a valgus position (Neumann, p. 619). With pronation there is also associated abduction of the forefoot (Magee, p. 896). Varum and adduction are incorrect. 3. The picture demonstrates a pes planus with foot pronation. The subtalar joint's main components with this are eversion and abduction (Neumann, p. 609). The midtarsal joint (the transverse tarsal joints) also everts (Neumann, p. 614). In stance, the calcaneus is excessively everted from midline, which is a valgus position (Neumann, p. 619). With pronation there is also associated abduction of the forefoot (Magee, p. 896). 4. The picture demonstrates a pes planus with foot pronation. The subtalar joint's main components with this are eversion and abduction (Neumann, p. 609). The midtarsal joint (the transverse tarsal joints) also everts (Neumann, p. 614). In stance, the calcaneus is excessively everted from midline, which is a valgus position (Neumann, p. 619). With pronation there is also associated abduction of the forefoot (Magee, p. 896). Varum and inversion are incorrect. Author: Neumann DATitle: Kinesiology of the Musculoskeletal System: Foundations for RehabilitationEdition: 3Publisher: ElsevierYear: 2017Pages: 609, 614, 619 Author: Magee DJTitle: Orthopedic Physical AssessmentEdition: 6Publisher: Elsevier SaundersYear: 2014Pages: 896

A patient is admitted to the hospital with a fever and acute hip pain. Radiographic evaluation reveals a lesion in the proximal femur. Which of the following procedures is LEAST helpful in making a differential diagnosis? 1. Tissue biopsy 2. Bone scan 3. Electromyogram 4. Blood tests

3 1. The radiographic findings are consistent with a bone condition. A biopsy is the definitive diagnostic procedure to identify bone tumors (p. 1257). 2. The radiographic findings are consistent with a bone condition. A bone scan may be used to identify osteopenia and osteoporosis (p. 1219). 3. The radiographic findings are consistent with a bone condition. Electromyogram (EMG) is more helpful for diagnosing neuromuscular disorders (p. 1665). 4. The radiographic findings are consistent with a bone condition. Various blood tests can diagnose and differentiate musculoskeletal neoplasms (p. 1257). Author: Goodman CC, Fuller KSTitle: Pathology: Implications for the Physical TherapistEdition: 4Publisher: Elsevier SaundersYear: 2015Pages: 1219, 1257, 1665

A physical therapist is treating a patient who has a Stage 3 pressure injury. The affected area has red granulation tissue, no eschar, no odor, and minimal serosanguineous drainage. Which of the following is the BEST treatment for this wound? 1. Thin, occlusive film, changed as needed 2. Wet-to-dry normal saline on gauze, changed 3 times/day 3. Hydrocolloid dressing, changed as needed 4. Pulsed lavage and sharp debridement

3 1. Thin, occlusive film is used mostly for cosmetic healing and will not absorb drainage. 2. Wet-to-dry normal saline on gauze should not be used with granulating tissue. 3. Hydrocolloids handle minimal to moderate exudate in noninfected wounds. 4. Debridement would not be needed for the wound because there is no eschar. Author: Myers BATitle: Wound Management: Principles and PracticeEdition: 3Publisher: PearsonYear: 2012Pages: 141

A physical therapist working with a patient who is borderline hypotensive can minimize orthostatic hypotension by: 1. loosening tight legwear and footwear before gait training. 2. elevating the head during a hypotensive episode. 3. instructing the patient to perform ankle pumps before standing. 4. encouraging the patient to consume meals prior to therapy.

3 1. Tight stockings can be used to reduce orthostatic hypotension. 2. The head of the bed should be lowered during hypotensive episodes. 3. Repeatedly dorsiflexing the feet (ankle pumps) can ameliorate symptoms of orthostatic hypotension. 4. Consuming meals before therapy will not affect orthostatic hypotension. Author: Goodman CC, Fuller KSTitle: Pathology: Implications for the Physical TherapistEdition: 4Publisher: Elsevier SaundersYear: 2015Pages: 601

Which of the following methods is BEST to assess a patient's passive range of motion of the shoulder? 1. The patient produces the movement by raising a cane with both upper extremities. 2. The patient produces the movement with assistance of the opposite upper extremity. 3. The physical therapist moves the extremity to the end of the available range. 4. The physical therapist moves the extremity beyond the tissue resistance.

3 1. Use of a cane and both upper limbs describes active assistive range of motion rather than passive (p. 52). 2. Producing the movement with assistance of the opposite upper extremity describes active assistive range of motion rather than passive (p. 52). 3. Passive range of motion is produced entirely by an external force. It should be performed within the available range. (p. 52) 4. Movement of an extremity beyond tissue resistance by the therapist is a stretching maneuver rather than a determination of the available passive range (p. 75). Author: Kisner C, Colby LATitle: Therapeutic Exercise: Foundations and TechniquesEdition: 6Publisher: F.A. DavisYear: 2012Pages: 52, 75

The medical record indicates that a patient has a deficiency of vitamin D. The patient is MOST at risk for developing which of the following conditions? 1. Fragile skin 2. Excessive bleeding 3. Bone decalcification 4. Poor vision

3 1. Vitamin D is not primarily involved in maintaining skin integrity. 2. Bleeding disorders are related to hematological disorders, not vitamin D deficiency. 3. Vitamin D is important for calcium absorption, synthesis, and transport, and bone decalcification can result from a deficiency. 4. Vitamin D is not primarily involved in maintaining proper vision. Author: Goodman CC, Fuller KSTitle: Pathology: Implications for the Physical TherapistEdition: 4Publisher: Elsevier SaundersYear: 2015Pages: 1026, 1218-1219

The parent of a child with myelomeningocele at L2 asks a physical therapist to provide information regarding the child's prognosis for walking. The therapist should respond by telling the parent that the child will MOSTlikely be able to: 1. walk with ankle-foot orthoses throughout the home and school. 2. walk with knee-ankle-foot orthoses throughout the home and school. 3. walk with knee-ankle-foot orthoses within the home. 4. stand using a parapodium; walking will not be possible.

3 1. Walking in the home and community using only ankle-foot orthoses would be possible for a child who has a L4-L5myelomeningocele, not a L2 lesion. 2. A child who has a L3-L4 myelomeningocele would be expected to achieve community ambulation using knee-ankle-foot orthoses, not a child with L2 myelomeningocele. 3. A child who has L1-L2 level myelomeningocele would be expected to walk short distances in the home. 4. Standing in a parapodium would be an appropriate prognosis for a child who has a thoracic-level myelomeningocele. A child who has an L2 myelomeningocele is expected to achieve ambulation over short distances with orthoses. Author: Palisano RJ, Orlin MN, Schreiber JTitle: Campbell's Physical Therapy for ChildrenEdition: 5Publisher: ElsevierYear: 2017Pages: 572-573

A nurse caring for a patient who is 2 days post cerebrovascular accident asks the physical therapist for positioning recommendations when the patient lies on the hemiplegic side. The therapist's recommendations should include positioning the: 1. wrist in a flexed position. 2. elbow in a flexed position. 3. scapula in a protracted position. 4. forearm in a pronated position.

3 1. When a patient lies on the hemiplegic side, the position of the wrist should be neutral, not flexed. 2. When a patient lies on the hemiplegic side, the elbow should be extended, not flexed. 3. When a patient lies on the hemiplegic side, the scapula of the hemiplegic arm should be protracted. This is the best choice. 4. When a patient lies on the hemiplegic side, the forearm should be supinated, not pronated. Author: O'Sullivan SB, Schmitz TJ, Fulk GDTitle: Physical RehabilitationEdition: 6Publisher: F.A. DavisYear: 2014Pages: 682

A 75-year-old individual engages in a daily aerobic exercise program. Which of the following gastrointestinal tract changes is MOST likely a result of the exercise? 1. Increased lactase levels 2. Increased bowel transit time 3. Improved neuromuscular function of the colon 4. Improved upper esophageal sphincter strength

3 1. With aging, lactase levels decrease, leading to intolerance of dairy products by many older adults (lactose intolerance). This is a common occurrence in the small intestine with aging and would not be affected by exercise. 2. Exercise has been demonstrated to decrease gastrointestinal (bowel transit) transit time, thus improving the overall efficiency of the gastrointestinal system. 3. Aerobic exercise has been found to improve the overall neuromuscular function of the colon, thus improving bowel transit time and decreasing the risk of constipation. 4. With age, the strength of esophageal contractions and the tension in the upper esophageal sphincter decrease, but the movement of food is not impaired by these changes. However, many older adults are likely to be affected by diseases that interfere with esophageal contractions. Author: Goodman CC, Fuller KSTitle: Pathology: Implications for the Physical TherapistEdition: 4Publisher: Elsevier SaundersYear: 2015Pages: 866-867

A physical therapist is preparing to perform a bed to chair transfer with a patient who underwent an extensive surgery 24 hours ago and is taking an angiotensin-converting enzyme (ACE) inhibitor. During this transfer, the physical therapist should monitor the patient for the possibility of a: 1. rise in systolic and a drop in diastolic blood pressure. 2. rise in systolic and diastolic blood pressure. 3. drop in systolic and diastolic blood pressure. 4. drop in systolic and a rise in diastolic blood pressure.

3 1. With orthostatic hypotension, there is a characteristic drop in both systolic and diastolic blood pressure, not a rise in systolic blood pressure. 2. With orthostatic hypotension, there is a characteristic drop in systolic and diastolic blood pressure, not a rise. 3. Orthostatic hypotension (a drop in systolic and often diastolic blood pressure) is a concern in those individuals who have been in supine position for a period of time and are taking antihypertensive agents. 4. With orthostatic hypotension, there is a characteristic drop in both systolic and diastolic blood pressure, not a rise in diastolic pressure. Author: O'Sullivan SB, Schmitz TJ, Fulk GDTitle: Physical RehabilitationEdition: 6Publisher: F.A. DavisYear: 2014Pages: 541 Author: Goodman CC, Snyder TEKTitle: Differential Diagnosis for Physical Therapists: Screening for ReferralEdition: 5Publisher: Elsevier SaundersYear: 2013Pages: 275, 277

A physical therapist wants to investigate the efficacy of iontophoresis in treatment of calcium deposits. Which of the following study designs provides the MOST conclusive cause-and-effect analysis on which to base a clinical decision? 1. Cohort study 2. Case-control study 3. Case series 4. Randomized controlled trial

4 1. A cohort study does not provide a control or comparison group to control for threats to internal validity. 2. A case-control study does not provide a control or comparison group to control for threats to internal validity. 3. A case series does not provide a control or comparison group to control for threats to internal validity. 4. Randomized controlled trial is the only study design that appropriately evaluates cause-effect for therapeutic efficacy. Author: Portney LG, Watkins MPTitle: Foundations of Clinical Research: Applications to PracticeEdition: 3Publisher: F.A. DavisYear: 2015Pages: 21

A patient with a right transtibial amputation is referred for prosthetic gait training. Examination of gait reveals lateral trunk bending toward the right side during right midstance. The MOST likely cause of this problem is that the: 1. foot is too far medially inset. 2. socket is in excessive adduction. 3. foot is too far posteriorly set. 4. prosthesis is too short.

4 1. A prosthetic foot that is too far medially inset would cause the prosthetic foot to lean laterally. 2. A socket in excessive adduction would cause the prosthetic foot to lean medially. 3. A prosthetic foot that is too far posteriorly set would cause early heel rise. 4. A prosthesis that is too short would cause lateral trunk bending toward the ipsilateral side during midstance on that side. Author: O'Sullivan SB, Schmitz TJ, Fulk GDTitle: Physical RehabilitationEdition: 6Publisher: F.A. DavisYear: 2014Pages: 1388

A physical therapist is setting up an exercise program for a patient who is interested in improving cardiovascular fitness. When performing a submaximal cycle ergometer test the therapist should expect a relatively constant value for: 1. oxygen consumption. 2. heart rate. 3. systolic blood pressure. 4. diastolic blood pressure.

4 1. An increase in oxygen uptake occurs in response to increased workload (O'Sullivan, p. 531). 2. Heart rate increases gradually in response to increased workload (Hillegass, p. 530). 3. Systolic blood pressure should increase with increasing workload by approximately 10 mm Hg per 1 metabolic equivalent (MET) increase in workload (Hillegass, pp. 532-533). 4. Diastolic blood pressure should remain relatively constant during exercise, remaining within 10 mm Hg of the starting point (Hillegass, p. 533). Author: O'Sullivan SB, Schmitz TJ, Fulk GDTitle: Physical RehabilitationEdition: 6Publisher: F.A. DavisYear: 2014Pages: 531 Author: Hillegass ETitle: Essentials of Cardiopulmonary Physical TherapyEdition: 4Publisher: ElsevierYear: 2017Pages: 530, 532-533

A patient who is being treated for fibromyalgia arrives for an outpatient physical therapy appointment with reports of fatigue, weakness, blurred vision, and excessive thirst. The physical therapist should suspect the presence of which of the following conditions? 1. Liver disease 2. Hypothyroidism 3. Cushing syndrome 4. Diabetes mellitus

4 1. Blurred vision and excessive thirst are not common symptoms of liver disease (p. 363). 2. Blurred vision and excessive thirst are not common symptoms of hypothyroidism (p. 420). 3. Blurred vision and excessive thirst are not common symptoms of Cushing syndrome (p. 416). 4. The symptoms listed are all characteristic of diabetes. Although fatigue and weakness are common in other conditions, the blurred vision and excessive thirst in combination should cause one to suspect diabetes over and above other conditions. (p. 426) Author: Goodman CC, Snyder TEKTitle: Differential Diagnosis for Physical Therapists: Screening for ReferralEdition: 5Publisher: Elsevier SaundersYear: 2013Pages: 363, 416, 420, 426

In a study of changes in muscle strength, several physical therapists performed manual muscle tests on volunteer subjects. The results of the tests are BESTcategorized as which of the following types of data? 1. Reliable data 2. Ratio data 3. Interval data 4. Ordinal data

4 1. Data from manual muscle tests should not be considered reliable unless the reliability of the test has been established. 2. Manual muscle test scores do not provide known or equal distances between scores, nor do they provide an absolute 0, so cannot be considered ratio data. 3. Manual muscle test scores do not provide known or equal distances between scores, so cannot be considered interval data. 4. Manual muscle tests scores are considered ordinal because they label strength measures in rank order but do not provide known or equal distances between the scores. Author: Portney LG, Watkins MPTitle: Foundations of Clinical Research: Applications to PracticeEdition: 3Publisher: F.A. DavisYear: 2015Pages: 68, 87

Which of the following interventions is BEST for the wound shown in the photograph? 1. Enzymatic debridement 2. Wet-to-dry dressing 3. Whirlpool 4. Foam dressing above medial mallelous red round wound

4 1. Enzymatic debridement is effective only if there is necrotic tissue present in the wound bed. The illustrated wound does not have necrotic tissue, and, therefore, enzymatic debridement is not the best intervention in this case. (p. 442) 2. A wet-to-dry dressing is considered mechanical debridement and is used to remove necrotic tissue. The illustrated wound does not include necrotic tissue, and, therefore, this is not the best intervention in this case. (p. 441) 3. A whirlpool is considered a mechanical debridement intervention and is used primarily for wound cleansing and debridement. The illustrated wound does not include necrotic tissue, and, therefore, this is not the best intervention in this case. (pp. 740-741) 4. A moist balanced wound environment is needed for optimal wound healing. The wound in the photograph is a venous ulcer and requires an absorbent dressing. Foam dressings are one of the recommended dressing types for red wounds with high exudate. (p. 519) Author: Sussman C, Bates-Jensen BTitle: Wound Care: A Collaborative Practice Manual for Health ProfessionalsEdition: 4Publisher: Lippincott, Williams & WilkinsYear: 2012Pages: 441-442, 519, 740-741

A physical therapist is documenting functional outcomes for a patient who has been treated for low back pain. Which of the following improvements is the MOST significant to report? 1. The patient's straight-leg test result has changed from 40° to 75°. 2. The patient reports a decrease in average daily pain from 7/10 to 3/10. 3. The patient's upper and lower abdominal strength ratings have improved from Poor (2/5) to Good (4/5). 4. The patient has improved daily work tolerance from 1.5 hours to 3.0 hours.

4 1. Hamstring muscle length is an impairment and not a measure of function and, unless directly related to some functional activity, cannot be reported as a functional outcome. 2. Although the reported change in pain is an impressive drop, pain is an impairment and not a measure of function and cannot be reported as a functional outcome unless directly related to some functional activity. 3. Abdominal strength is an impairment and not a measure of function and, unless directly related to some functional activity, cannot be reported as a functional outcome. 4. Although the improvement in work tolerance is not large, of the choices, this option is the only one that relates to the patient's function and can be reported as a functional outcome. Author: Kisner C, Colby LATitle: Therapeutic Exercise: Foundations and TechniquesEdition: 6Publisher: F.A. DavisYear: 2012Pages: 25

A physical therapist testing a patient for ideomotor apraxia should have the patient: 1. state where her limb is in different positions in space. 2. select an object from an array of similarly shaped objects. 3. touch her nose, then touch a target 2 ft (.6 m) away. 4. sign her name as part of an activity and on command.

4 1. Identifying where the limb is in space is a test of joint position sense, not apraxia. 2. Selecting an object from an array of similarly shaped objects is a test of figure-ground discrimination. A positive result would indicate a perceptual dysfunction, which suggests a spatial relations syndrome. 3. Touching the nose then a target 2 ft (.6 m) away is the unilateral finger-to-nose test, which is a test of coordination and motor function, not perceptual function. 4. Ideomotor apraxia is the inability to perform purposeful movements when there is no loss of sensation, strength, coordination, or comprehension. Frequently, patients who have ideomotor apraxia will be unable to perform a motor task on command but will be able to perform the task when they are left on their own. Author: Shumway-Cook A, Woollacott MHTitle: Motor Control: Translating Research into Clinical PracticeEdition: 5Publisher: Wolters KluwerYear: 2017Pages: 540-541

Which of the following methods is the MOST effective way to encourage a patient who is learning a new movement pattern? 1. Describe how incorrect attempts differ from the correct movement before proceeding with the next trial. 2. Describe each component of the desired movement before beginning practice. 3. Provide feedback at the same point in each of the patient's attempts. 4. Provide frequent feedback during the initial trials of the movement.

4 1. Incorrect attempts corrected early may reduce the ability to learn a task. 2. Explanation of each component of movement may cause confusion during early learning of a motor task. 3. Providing feedback at the same point is not as effective as frequent feedback during early learning. 4. Early in learning, frequent feedback is easily brought to conscious attention and is important. Learning may be delayed if feedback is given too infrequently during the early stages of learning a movement pattern. Author: O'Sullivan SB, Schmitz TJ, Fulk GDTitle: Physical RehabilitationEdition: 6Publisher: F.A. DavisYear: 2014Pages: 410, 412

During a physical therapy examination, a patient reports numbness, burning, and paresthesias in both feet. A neurological examination reveals bilateral loss of pain and temperature sensation in a stocking-and-glove distribution, diminished ankle reflexes, normal knee-jerk reflexes, and decreased strength in the feet and ankles bilaterally. The patient MOST likely has: 1. L4-L5 nerve root compression. 2. intermittent claudication. 3. peripheral vascular disease. 4. peripheral neuropathy.

4 1. L4-L5 nerve root compression would cause sensory impairments in a dermatomal pattern (p. 125). 2. Intermittent claudication is a vascular problem that causes pain, but no sensory or motor loss (p. 244). 3. Peripheral vascular disease would not cause sensory changes or altered reflexes (p. 569). 4. A stocking-and-glove distribution of sensory impairment and distal/peripheral loss of motor function are classic signs of peripheral neuropathy (p. 469). Author: Goodman CC, Snyder TEKTitle: Differential Diagnosis for Physical Therapists: Screening for ReferralEdition: 5Publisher: Elsevier SaundersYear: 2013Pages: 125, 244, 469, 569

A patient with C3 spinal cord injury is working with a physical therapist to select an appropriate power wheelchair. The wheelchair that the patient is trying out has the following features: power-elevating leg rests, power recline, and chest strap. When the patient performs pressure relief by reclining the wheelchair, spasticity increases in the patient's lower extremities. The patient should try a wheelchair with: 1. lateral hip guides. 2. non-elevating leg rests. 3. foot straps. 4. power tilt.

4 1. Lateral hip guides will not provide pressure relief. 2. Non-elevating leg rests will not provide pressure relief or address the patient's spasticity. 3. Foot straps would fixate the feet and could cause injury to the patient if the patient reclined and spasticity increased. 4. A wheelchair with power tilt provides pressure relief and is less likely to elicit spasticity than a wheelchair with power recline. Author: Umphred DA, Lazaro RT, Roller ML, Burton GUTitle: Umphred's Neurological RehabilitationEdition: 6Publisher: Elsevier MosbyYear: 2013Pages: 505

Which of the following exercise guidelines would be BEST for an otherwise healthy adult patient undergoing radiation treatment for breast cancer? 1. Postpone exercise until 1 month after radiation treatments end. 2. Initiate exercise activities 1 hour after radiation treatments. 3. Walk on a treadmill at a level of 6/20 on the Borg scale. 4. Walk on a treadmill at a moderate level of intensity.

4 1. Low to moderate intensity exercise during the weeks of radiation treatment can help manage treatment-related symptoms such as fatigue. Ideally exercise should take place during the period when treatment sessions are being conducted. (p. 181) 2. Current guidelines recommend clients should be advised not to exercise within 2 hours of radiation therapy, because an increase in circulation with exercise may increase the effects of these treatments (p. 181). 3. A level of 6/20 is no exertion and would not be of benefit. Patients should exercise at a rating of perceived exertion not to exceed 15 to 17 for moderate intensity training. (pp. 180-181) 4. Radiation therapy can cause scarring to the heart and lungs. Aerobic exercise should be kept at a low to moderate intensity. (p. 181) Author: Goodman CC, Fuller KSTitle: Pathology: Implications for the Physical TherapistEdition: 4Publisher: Elsevier SaundersYear: 2015Pages: 180-181

A patient has a small, hyper-irritable nodule at the superior medial border of the left scapula. Pressure on this nodule causes the patient to experience pain in the left lateral cervical area from C2-C7 and at the medial border of the left scapula. Which of the following interventions is MOST appropriate for this condition? 1. Intermittent, mechanical traction with 20 lb (9.1 kg), with the cervical spine in a neutral position 2. Posterior-anterior glides of C2-C7 performed into tissue resistance 3. Instruction in home cervical spine stabilization exercises 4. Spray and stretch to the left levator scapulae muscle

4 1. Mechanical traction would be indicated for cervical impingement or hypomobility. The patient has signs and symptoms of trigger point involvement of the left levator scapulae. 2. Joint mobilization would be appropriate for hypomobility, which was not described in the patient. 3. Stabilization exercise would be more appropriate to address instability. The patient has trigger point involvement in the left levator scapulae. Instability was not described. 4. The signs and symptoms are consistent with a trigger point in the left levator scapulae; one appropriate treatment for this condition is spray and stretch to the affected muscle. Author: Dutton MTitle: Dutton's Orthopaedic Examination, Evaluation, and InterventionEdition: 4Publisher: McGraw-HillYear: 2017Pages: 440-441

A 21-year-old female ballet dancer who has had an insidious onset of pelvic and hip pain is referred to physical therapy. During the history the patient reports relief of symptoms after passing gas. The patient exhibits full, pain-free hip and lumbar range of motion and normal lower extremity strength. The patient's pain is unchanged by walking or lying supine. Which of the following causes of the pain is MOST likely? 1. Pancreatitis 2. Lumbar herniated nucleus pulposus 3. Sacroiliac dysfunction 4. Crohn disease

4 1. Pancreatitis will typically be made worse by walking or lying in supine position (pp. 341-342). 2. A patient who has a herniated nucleus pulposus will typically have loss of lumbar motion and decreased symptoms with lying supine in position (p. 110). 3. Sacroiliac dysfunction will typically cause decreased hip or lumbar range of motion and will not be affected by the passing of gas (p. 110). 4. Crohn disease can refer pain to the pelvic area and hip that is relieved by passing gas (p. 344). Author: Goodman CC, Snyder TEKTitle: Differential Diagnosis for Physical Therapists: Screening for ReferralEdition: 5Publisher: Elsevier SaundersYear: 2013Pages: 110, 341-342, 344

Which of the following conditions is an absolute CONTRAINDICATION to a patient's participation in aquatic physical therapy? 1. Advanced peripheral neuropathy 2. Chronic venous insufficiency 3. History of seizure disorder 4. Severe kidney disease

4 1. Patients who have advanced peripheral neuropathy such as patients with heat intolerant multiple sclerosis, may become fatigued with immersion in temperatures greater than 94.1°F (33°C) (p. 291). Therapeutic exercise performed in water is typically done in water that is warmer than 94.1°F (33°C) (p. 295). Patients can still participate in aquatic therapy even though they are fatigued as long as they are closely monitored. 2. Small open wounds [from chronic venous insufficiency] may be covered by waterproof dressings (p. 292). 3. Patients who have controlled epilepsy require close monitoring during immersed treatment and must be adherent with medication prior to treatment (p. 292). 4. Patients who have severe kidney disease will be unable to adjust to fluid loss during immersion (p. 292). Author: Kisner C, Colby LATitle: Therapeutic Exercise: Foundations and TechniquesEdition: 6Publisher: F.A. DavisYear: 2012Pages: 291-292, 295

Standing postural assessment of a patient with spondylolysis reveals an increased lumbar lordosis, shortened hip flexors, and Fair (3/5) strength in the abdominal muscles. Which exercises would be MOSTappropriate for this patient? 1. In prone, perform press-ups, raising upper trunk with arms while keeping pelvis in contact with table. 2. Chair-sitting with hips and knees flexed to 90°, perform alternate anterior and posterior pelvic tilts. 3. In supine, perform straight-leg raises to stretch out hamstring muscles. 4. In supine with knees flexed, perform posterior pelvic tilts with isometric contraction of rectus abdominis.

4 1. Performing prone press-ups would further increase the lumbar lordosis. Lumbar extension should be avoided. 2. This position may allow the short hip flexors to contribute to the movement without strengthening the weak abdominal muscles. Because the abdominal muscles are significantly weak, only posterior tilts performed with isometric rectus contraction will prevent strengthening of the hip flexors instead of the abdominal muscles. 3. When the abdominal muscles are weak, straight leg raises result in back hyperextension and possibly pain and, therefore, should be avoided. 4. When the hip flexors are short, the knees should be flexed. Because the abdominal muscles are significantly weak, only posterior tilts performed with isometric rectus contraction will prevent strengthening of the hip flexors instead of the abdominal muscles. Author: Brody LT, Hall CMTitle: Therapeutic Exercise: Moving Toward FunctionEdition: 4Publisher: Wolters KluwerYear: 2018Pages: 479

Which of the following strategies is MOST effective for helping a patient with limited recall learn to do three exercises independently? 1. Allow the patient to perform the exercises through a partial range of motion. 2. Have the patient complete the exercises in a group setting. 3. Decrease the goal to one exercise done with supervision. 4. Gradually reduce the number of verbal cues.

4 1. Practice of parts may help with complex tasks, but practice of the full task with fading feedback is more important for learning. 2. Having the patient complete the exercises in a group setting may be distracting to an individual with limited recall. 3. Decreasing the goal to one exercise does not meet the goal of learning three exercises. 4. Gradually reducing the feedback (fading schedule) forces the subject to use internal processes of error detection and results in better delayed retention. Author: Shumway-Cook A, Woollacott MHTitle: Motor Control: Translating Research into Clinical PracticeEdition: 5Publisher: Wolters KluwerYear: 2017Pages: 33

Which of the following conditions must be ruled out when a physical therapist evaluates a patient with nonspecific neck pain? 1. Kidney stones 2. Pancreatic cancer 3. Colon cancer 4. Myocardial infarction

4 1. Pyelonephritis would refer pain to the thoracic or lumbar areas. 2. Pancreatic cancer would refer pain to the thoracic or lumbar areas. 3. Colon cancer would refer pain to the lumbar or sacral areas. 4. Myocardial infarction is a viscerogenic cause of nonspecific neck pain. Author: Goodman CC, Snyder TEKTitle: Differential Diagnosis for Physical Therapists: Screening for ReferralEdition: 5Publisher: Elsevier SaundersYear: 2013Pages: 547

A patient reports multiple myalgias, fatigue, weight gain despite decreased food intake, and frequently feeling cold. The physical therapist should expect information from which of the following tests to be MOST helpful in managing the patient's care? 1. Rheumatoid factor 2. C-reactive protein 3. Fasting blood glucose level 4. Thyroid stimulating hormone level

4 1. Rheumatoid factor is appropriate for determining the presence of rheumatoid arthritis or other inflammatory conditions. A patient who has rheumatoid arthritis would be more likely to report arthralgias than myalgias. (p. 452) 2. The patient is describing symptoms of hypothyroidism. C-reactive protein is a nonspecific indicator of inflammation or infection. It wouldn't provide the most pertinent information with this diagnosis. (pp. 248, 465) 3. Fasting blood glucose levels determine the amount of sugar (glucose) in the blood. This is an appropriate test for diabetes. Fatigue and weight loss are associated with diabetes; however, polyuria and polydipsia are often reported. The patient's report of myalgias and impaired thermoregulation is more consistent with hypothyroidism. (pp. 425-426) 4. The patient is describing symptoms of hypothyroidism. When hypothyroidism is present, the blood levels of thyroid hormones can be measured directly and are decreased. The main tool for the detection of thyroid disease is the measurement of thyroid stimulating hormone. (pp. 417, 420) Author: Goodman CC, Snyder TEKTitle: Differential Diagnosis for Physical Therapists: Screening for ReferralEdition: 5Publisher: Elsevier SaundersYear: 2013Pages: 248, 417, 420, 425-426, 452, 465

A child with myelomeningocele and a history of hydrocephalus begins to exhibit irritability, lethargy, and vomiting. Which of the following is the MOST likely cause of these symptoms? 1. Tethered cord 2. Urinary tract infection 3. Arnold-Chiari malformation 4. Shunt malfunction

4 1. Signs and symptoms of tethered cord include changes in bowel and bladder function, increased spasticity, back pain. Tethered cord is not associated with irritability, lethargy, and vomiting (Umphred, p. 423). 2. Signs and symptoms of urinary tract infection include urinary frequency, dysuria, and pyuria. Urinary tract infection is not associated with lethargy and vomiting (Paz, p. 227). 3. Signs and symptoms of Arnold-Chiari malformation are weakness, pain, sensory changes, vertigo, diplopia, and ataxia. Arnold-Chiari malformation is not associated with irritability, lethargy, and vomiting (Palisano, p. 556). 4. Irritability, lethargy, and vomiting are all signs and symptoms of shunt malfunction (Palisano, p. 554). Author: Palisano RJ, Orlin MN, Schreiber JTitle: Campbell's Physical Therapy for ChildrenEdition: 5Publisher: ElsevierYear: 2017Pages: 554, 556 Author: Umphred DA, Lazaro RT, Roller ML, Burton GUTitle: Umphred's Neurological RehabilitationEdition: 6Publisher: Elsevier MosbyYear: 2013Pages: 423 Author: Paz JC, West MPTitle: Acute Care Handbook for Physical TherapistsEdition: 4Publisher: Elsevier SaundersYear: 2014Pages: 227

A patient had an arthroscopic patellar debridement and lateral retinacular release of the knee 8 weeks ago. The patient works as a computer programmer, has two small children, and lives in a three-story home. Which of the following information is MOST important for determining whether the patient is appropriate for discharge from outpatient therapy? 1. Single leg hop for distance 2. Isokinetic strength comparison of the quadriceps and hamstrings 3. Passive knee flexion range of motion 4. Unilateral step down from an 8-inch (20.3-cm) stair

4 1. Single leg hop for distance doesn't determine how well the patient can negotiate stairs or squat, which might be required for attending to small children. 2. Although isokinetic strength may also be of interest, it may not provide answers about functional tolerances. Simply because the patient may test strong with isokinetic testing does not mean the patient will be able to resume previous activities such as stair climbing. 3. Passive knee motion would be expected to be normal in a patient 8 weeks after arthroscopic patellar debridement and lateral retinacular release. Range of motion is not sufficient to determine discharge; functional outcomes would be better. 4. The patient would need to be proficient with stair climbing due to the living situation, and stair climbing likely was problematic due to the nature of the surgery. Unilateral step downs most closely match home activities/activities of daily living (ADLs) and is a meaningful activity for the patient. Author: Kisner C, Colby LATitle: Therapeutic Exercise: Foundations and TechniquesEdition: 6Publisher: F.A. DavisYear: 2012Pages: 772-773, 791

Which of the following skin conditions may have an associated arthritic component? 1. Stasis dermatitis 2. Seborrheic keratosis 3. Herpes zoster 4. Psoriasis

4 1. Stasis dermatitis is a result of venous insufficiency. It is not associated with arthritis. (p. 424) 2. Seborrheic keratosis is a benign tumor of the skin that is usually associated with hormonal therapy or inflammatory dermatoses (p. 431). 3. Herpes zoster is a viral infection of a nerve root with associated vesicular eruption of the skin in the nerve's dermatome. This condition is associated with postherpetic neuralgia, not arthritis. (pp. 351-352) 4. Approximately 10% of people with psoriasis develop arthritic symptoms referred to as psoriatic arthritis (pp. 442-443). Author: Goodman CC, Fuller KSTitle: Pathology: Implications for the Physical TherapistEdition: 4Publisher: Elsevier SaundersYear: 2015Pages: 351-352, 424, 431, 442-443

A patient with a recent total knee arthroplasty and a new diagnosis of hiatal hernia is concerned about the exercise program. Which of the following responses by the physical therapist would be MOST appropriate? 1. Supine exercises are best for patients with a hiatal hernia. 2. Exercise is contraindicated for patients with a hiatal hernia. 3. There are no restrictions on exercise for patients with a hiatal hernia. 4. The Valsalva maneuver should be avoided during exercise.

4 1. Supine position can cause the lower esophagus and stomach to be pulled into the thorax, thus increasing symptoms or discomfort. Although not contraindicated, these exercises would not be best. (p. 1429) 2. Exercise can help with some of the risk factors for hernia, including obesity/weight control (p. 1430). 3. The physical therapist should educate the patient on the challenges of supine exercise, recumbent exercises, exercises that involve bending, and exercises that increase abdominal pressure, exacerbating symptoms (pp. 1429-1430). 4. Patients who have a hiatal hernia should avoid the Valsalva maneuver. The Valsalva maneuver increases intraabdominal pressure, which can worsen the hernia (pp. 1429-1430). Author: McCance KL, Huether SE, Brashers VL, Rote NSTitle: Pathophysiology: The Biologic Basis for Disease in Adults and ChildrenEdition: 7Publisher: Elsevier MosbyYear: 2014Pages: 1429-1430

A patient has a dry, yellow wound located on the anterolateral aspect of the left lower extremity. The patient experiences pain when walking continuously for 5 minutes and therefore avoids walking. Which of the following home exercise programs is MOST appropriate for the patient? 1. Swimming in a heated therapeutic pool 2. Upper extremity progressive resistive exercise program 3. Lower extremity progressive resistive exercise program 4. Intermittent walking program to tolerance

4 1. Swimming is an appropriate exercise medium for patients who have arterial insufficiency, as it is a non-weight-bearing activity. However, it is contraindicated for a patient with an open wound. 2. Upper extremity exercises are an appropriate treatment intervention for patients who have arterial disease; however, upper extremity exercises would not address the need to increase blood supply to the lower extremity and, therefore, would not be the best response. 3. A progressive resistive exercise program addresses the need to increase strength, but the patient described would be in more need of an intervention to improve circulation and muscular endurance, rather than strength. 4. Periods of brief walking can increase oxygenation in the feet and limbs of patients who have arterial disease. The presence of a wound would not preclude the recommendation of a walking program for this patient and, therefore, would be beneficial for the patient. Walking is the most functional mode of exercise to complete at home. Author: O'Sullivan SB, Schmitz TJ, Fulk GDTitle: Physical RehabilitationEdition: 6Publisher: F.A. DavisYear: 2014Pages: 585-586, 624

A newborn's Apgar score represents evaluation of heart rate, color, and which additional clinical characteristics? 1. Respiration, temperature, and weight 2. Reflex irritability, temperature, and weight 3. Respiration, muscle tone, and weight 4. Respiration, muscle tone, and reflex irritability

4 1. Temperature and weight are not parameters of physiological function assessed with the Apgar score. 2. Temperature and weight are not parameters of physiological function assessed with the Apgar score. 3. Weight is not a parameter used in assigning an Apgar score. 4. A newborn's Apgar score is based on these five parameters: heart rate, color, respiration, muscle tone, and reflex irritability. Author: Title: Taber's Cyclopedic Medical DictionaryEdition: 22Publisher: F.A. DavisYear: 2013Pages: 167-168

A physical therapist is performing passive range of motion of all extremities with a patient in the intensive care unit who is in a coma. The therapist notes the electrocardiogram in the photograph. The physical therapist's INITIALresponse should be to: 1. continue with present treatment. 2. continue passive range of motion of lower extremities only. 3. stop the treatment and monitor electrocardiogram for 5 minutes. 4. activate the emergency system. ventricular fibrillation

4 1. The electrocardiogram shows ventricular fibrillation, which is a medical emergency. Continuing treatment is inappropriate. 2. The electrocardiogram shows ventricular fibrillation, which is a medical emergency. Continuing treatment is inappropriate 3. The electrocardiogram shows ventricular fibrillation, which is a medical emergency. Medical attention should be sought immediately, not delayed for 5 minutes. 4. The electrocardiogram shows ventricular fibrillation, which is a medical emergency. Author: Frownfelter D, Dean ETitle: Cardiovascular and Pulmonary Physical Therapy: Evidence to PracticeEdition: 5Publisher: Elsevier MosbyYear: 2012Pages: 169

A physical therapist determines that a patient would benefit from vestibular rehabilitation, which is an area beyond the therapist's expertise. Which of the following courses of action is MOST appropriate for the therapist? 1. Discharge the patient and refer the patient back to the physician. 2. Inform the patient that vestibular rehabilitation is beyond the scope of physical therapy practice. 3. Seek treatment advice from a qualified provider without disclosure to the patient. 4. Inform the patient of the findings and assist in locating a qualified provider.

4 1. The patient does not need to be discharged, but instead can be referred to an appropriate practitioner. 2. Vestibular rehabilitation is within the scope of physical therapist practice. 3. The patient should be informed about the physical therapist's limited scope of knowledge. 4. A physical therapist should inform a patient if the needed interventions are outside the scope of the physical therapist's knowledge and should refer the patient to an appropriate practitioner. Author: Purtilo R, Haddad A, Doherty RTitle: Health Professional and Patient InteractionEdition: 8Publisher: Elsevier SaundersYear: 2014Pages: 91 Author: Pagliarulo MATitle: Introduction to Physical TherapyEdition: 5Publisher: ElsevierYear: 2016Pages: 233

During warm-up on a stationary bike, a patient reports feelings of fatigue. Following the activity, the physical therapist observes the patient sitting in a chair with elbows resting on the knees. The physical therapist should examine the patient for which of the following? 1. Trunk muscle weakness 2. Poor sitting posture 3. Lower extremity weakness 4. Use of accessory muscles for breathing

4 1. The preferred posture after the activity reflects the patient's need to improve breathing and minimize fatigue. It is not an indication of trunk muscle weakness. 2. The preferred posture after the activity reflects the patient's need to improve breathing and minimize fatigue. It is not an indication of poor sitting posture. 3. The preferred posture after the activity reflects the patient's need to improve breathing and minimize fatigue. It is not an indication of lower extremity weakness. 4. Fatigue, which was evident during warm-up, may have worsened after the activity. The patient's preferred position after the activity increases the effectiveness of the pectoralis and serratus anterior muscles to act as accessory muscles of inspiration by fixing their insertions, thereby improving breathing and minimizing fatigue. Author: Hillegass ETitle: Essentials of Cardiopulmonary Physical TherapyEdition: 4Publisher: ElsevierYear: 2017Pages: 6-7, 190, 514

While pivoting on a planted foot with an extended knee, an athlete feels a pop and falls to the ground. Upon standing, the athlete reports that the knee feels unstable. Which of the following tests is MOST likely to detect the pathological condition of the knee? 1. Varus stress test 2. McMurray test 3. Patellar compression test 4. Lachman test

4 1. The varus stress test is for a lateral collateral ligament problem. The mechanism of injury described in the stem does not indicate a lateral collateral ligament problem. (pp. 1002-1003) 2. The McMurray test is used to evaluate menisci, and the patient's history, specifically the lack of stability, does not point to a meniscal injury (pp. 1010, 1051). 3. The patellar compression test is used to evaluate the patellofemoral joint, and the mechanism of injury described in the stem does not indicate a patellofemoral problem (p. 1040). 4. The Lachman test is used to evaluate the integrity of the anterior cruciate ligament. The patient's history indicates damage to the anterior cruciate ligament. (pp. 1003, 1046) Author: Dutton MTitle: Dutton's Orthopaedic Examination, Evaluation, and InterventionEdition: 4Publisher: McGraw-HillYear: 2017Pages: 1002-1003, 1010, 1040, 1046, 1051

Which of the following words would MOST likely describe neurogenic pain? 1. Throbbing 2. Aching 3. Agonizing 4. Burning

4 1. Throbbing pain is usually vascular. 2. Aching pain is usually musculoskeletal. 3. Agonizing pain can be emotional. 4. Burning is a common descriptor of neurogenic pain. Author: Goodman CC, Snyder TEKTitle: Differential Diagnosis for Physical Therapists: Screening for ReferralEdition: 5Publisher: Elsevier SaundersYear: 2013Pages: 104

During treatment with continuous ultrasound over the upper trapezius at 1 MHz frequency with 1.25 W/cm2, a patient reports increased pain and a burning sensation under the ultrasound head. Which of the following courses of action is MOST appropriate for the physical therapist? 1. Discontinue the treatment and call the ultrasound supplier. 2. Use a different ultrasound head. 3. Change to pulsed mode. 4. Decrease intensity.

4 1. Treatment may be appropriately modified by decreasing the intensity and does not need to be discontinued. 2. Changing to a pulsed mode would not create a thermal effect with the ultrasound treatment, which is the purpose of continuous ultrasound. Changing the intensity first would be more appropriate. 3. Pulsed mode would be less therapeutic than continuous mode; therefore, decreasing the intensity with continuous mode is more appropriate. 4. Because the treatment site is soft tissue and not a bony prominence, the symptoms would not be persistent pain. Also, the 1.25 W/cm2 setting is an appropriate setting. Therefore, reducing the intensity is an appropriate modification. Author: Cameron MHTitle: Physical Agents in Rehabilitation: From Research to PracticeEdition: 4Publisher: Elsevier SaundersYear: 2013Pages: 187

A physical therapist is teaching a patient wheelchair transfers. Which of the following techniques is MOSTappropriate for the physical therapist to employ to facilitate the patient's ability to generalize the learning? 1. Provide verbal feedback immediately after each transfer is completed. 2. Practice transfers back and forth between the bed and wheelchair repeatedly. 3. Encourage the patient to perform transfers at a consistent rate with the wheelchair situated at a set angle. 4. Vary the speed of the transfers and practice transfers in the bathroom, at the raised mat, and in the patient's room.

4 1. Verbal cues can be used to provide knowledge of performance, but should be faded with time (p. 153). 2. Practice transfers back and forth between the bed and wheelchair repeatedly does not include practicing the skill in a variety of environments or at various speeds, therefore, it is not the best choice to facilitate a patient's ability to generalize learning (pp. 154, 156-157). 3. Encouraging the patient to perform transfers at a consistent rate with the wheelchair situated at a set angle does not include practicing the skill in a variety of environments or at various speeds, therefore, it is not the best choice to facilitate a patient's ability to generalize learning (pp. 154, 156-157). 4. Practicing at various speeds and in a variety of environments facilitates functional learning, generalization of the task, and adaptation to more complex situations (pp. 154, 156-157). Author: O'Sullivan SB, Schmitz TJTitle: Improving Functional Outcomes in Physical RehabilitationEdition: 2Publisher: F.A. DavisYear: 2016Pages: 153-154, 156-157


संबंधित स्टडी सेट्स

Survey of World History A - Unit 6

View Set

AWS Certified Solutions Architect Chapter Exams

View Set

CompTIA Linux+ (XKO-004) Post-Assessment Quiz

View Set

Сетевые протоколы

View Set